Логические задачи с ответами 1 класс: Логические задачи для 1 класса по математике с ответами и решениями

Содержание

Сборник математических и логических задач для 1–4 классов. Электронная книга

Описание В книге представлены 10 готовых комплектов заданий, разработанных под руководством Марины Анатольевны Поповой – педагога высшей квалификационной категории с более чем 20-летним стажем, неоднократным победителем конкурсов профессионального мастерства педагогов, имеющей награды комитета образования администрации Псковской области.

Задания можно использовать для проведения олимпиад и других занятий по решению логических задач и задач на внимательность для учеников 1–4 класса в качестве внеурочной деятельности педагога. Также они могут быть использованы в качестве подготовки к участию в районных и городских олимпиадах, для проведения контрольных, проверочных работ, а также для тестирования учеников. Также задания годятся для самостоятельной подготовки учеников в домашних условиях.
Книга будет интересна педагогам, руководителям кружков и факультативов, а также активным родителям.

Задания представлены в виде теста – 10 вопросов и по 5 вариантов ответов к каждому вопросу.

Необходимо выбрать и указать только 1 правильный ответ в каждом вопросе. Данная форма тестирования широко применяется при оценке знаний учеников и, с учётом развития компьютерных технологий в обучении, будет применяться ещё шире. В конце книги приведены ответы, позволяющие легко проверить работы.

Задания предложены в форме, которая позволяет раздать готовые материалы для работы в классе или в качестве домашнего задания.

Содержание издания
1. Веселые олимпиадные и развивающие задания по математике для 1 класса.
2. Логические задания по математике для 1 класса.

3. Сказочные олимпиадные задания по математике для 1 класса.
4. Логические и развивающие задания по математике для 2 класса.
5. Логические задания по математике для 2 класса.
6. Развивающие задания по математике для 3 класса.
7. Олимпиадные задания с величинами по математике для 3 класса.
8. Исторические задачи по математике для 3 класса.
9. Исторические развивающие задания по математике для 4 класса.
10. Логические задания по математике для 4 класса.

Технические характеристики

Книга представлена в электронном виде в формате .pdf. Данный тип файлов открывают все браузеры и электронные книги. Также можно использовать программу Acrobat Reader или аналог.

Оплата и доставка

– Методы оплаты
Вы можете выбрать наиболее удобный для Вас способ оплаты. Интернет-магазин «Интеграл» предлагает Вам следующие варианты оплаты:
  • Банковские карты.
  • Интернет-банкинг – онлайн платежи.
  • Терминалы оплаты.
  • Банковские переводы.
  • Электронные деньги.
Более подробнее о методах оплаты.
– Доставка
Электронная доставка бесплатная. Ссылка на скачивание высылается на e-mail заказчика после оплаты.
На текущий момент мы не пересылаем покупателям коробочные версии или программы, записанные на CD или DVD носителях.
По всем вопросам обращайтесь на наш контактный e-mail: [email protected].

Отзывы покупателей о программе

Отзывов пока нет!

Логические задачи для 4 класса

Задача 1
Брату и сестре 2 года назад вместе было 15 лет. Сейчас сестре 13 лет.
Сколько должно пройти лет, чтобы брату исполнилось 9 лет?

Ответ:    3 года

Задача 2
Запиши число 7 при помощи четырех троек и знаков действий.
Найди несколько решений.

Ответ:    (7 = 3 : 3 + 3 + 3, 7 = 3 + 3 + 3 : 3, 7 = 3 + 3 : 3 + 3)

Задача 3
Речь пойдёт про единицы времени. Что можно узнать, данным произведением 60 х 60 х 24 х 7?

Ответ:    Количество секунд в неделю

Задача 4
На пароме помещается или 6 грузовиков, или 10 легковушек.
В четверг паром, полностью загруженный, 5 раз пересек реку и переправил 42 машины.
Сколько было среди них грузовиков?

Ответ:    12

Задача 5
В гости к Игорю пришли друзья.
Сколько их было, если каждый из них сложил из даты своего рождения число и номер месяца и получил 35? Причём даты рождения у всех гостей разные.

Ответ:    8

Задача 6
Ребята измеряли шагами длину игровой площадки.
У Лизы получилось 25 шагов, у Полины – 27, у Максима – 22, а у Юры – 24.
У кого из ребят самый короткий шаг?

Ответ:    У Полины

Задача 7
У сороконожки 90 ножек. Она купила 13 пар сапожек. Но при этом 16 ног остались босыми.
Сколько пар старых сапожек было на сороконожке до покупки новых сапожек?

Ответ:    24

Задача 8
Из 64 маленьких кубиков составили большой куб. Синей краской покрасили пять граней большого куба.
Назови количество маленьких кубиков с тремя синими гранями.

Ответ:    4 – по углам

Задача 9
Расставь скобки так, чтобы получилось верное равенство 211 – 126 – 74 · 8 = 88

Ответ:    (211 – 126 – 74) · 8 = 88

Задача 10
Если самое большое трехзначное число уменьшить на самое большое двузначное число, полученный результат разделить на 4, а затем вычесть 25, то получится возраст мудреца-звездочета.
Сколько лет звездочету?

Решение:

1. 999 — 99 = 900
2. 900 : 4 = 225
3. 225 — 25 = 200

Ответ: 200 лет

Задача 11
Длина прямоугольного бассейна в 5 раз больше его ширины, причем ширина на 20 м меньше. Найдите площадь дна бассейна.

Решение:
1. 20 : 4 = 5 (м) составляет 1 часть, и является шириной бассейна
2. 5 * 5 = 25 (м) длина бассейна
3. 25 * 5 = 125 (м2) площадь дна бассейна

Ответ: площадь бассейна равна 125 м2

Задача 12
Степа Смекалкин задумал число. Потом он уменьшил это число на 19 и к произведению прибавил 19. В ответе у него тоже получилось 19. Какое число задумал Степа?

Ответ:    0


         Дальше:    Задачи на логику 4 класс с решением

Занятие 1 Задачи на смекалку

КОНКУРС СМЕКАЛКИ Дорогой друг!

КОНКУРС СМЕКАЛКИ — 2018 Дорогой друг! Мы подготовили для тебя несколько заданий на смекалку, сообразительность и внимание. Распечатай, реши и сдай оформленные ответы при входе во Дворец на праздник Неделя

Подробнее

Логика в математике для дошкольников.

Логика в математике для дошкольников. * В комнате 4 угла. В каждом углу сидела кошка, напротив каждой кошки 3 кошки. Сколько кошек находилось в комнате? (4 кошки) * Как в решете воды принести? ( Воду можно

Подробнее

Пояснительная записка

Пояснительная записка Все задания предполагают творческое применение программных знаний, умений и навыков по данным предметам. Материалы данной работы могут быть использованы учителем при подготовке к

Подробнее

Внимание. Ответ: Ответ:

Задание 1. «Отыщи поросят» Внимание. Внимательно посмотрите на картинку. Отыщите всех поросят на полянке. Каждого поросёнка, которого найдёте, зачеркните карандашом. (вписать цифрой число зачёркнутых поросят)

Подробнее

МАТЕМАТИЧЕСКАЯ ВИКТОРИНА 7 КЛАСС

МАТЕМАТИЧЕСКАЯ ВИКТОРИНА 7 КЛАСС Цели: развитие математических способностей, сообразительности, любознательности, логического мышления; укрепление памяти учащихся; развитие познавательной активности; развитие

Подробнее

«Подвижные игры для работы с учащимися»

«для работы с учащимися» Выполнила: Воробьева Валентина Федоровна учитель биологии ГОУ СОШ 750 СВАО, педагог дополнительного образования, руководитель творческого объединения «Юный эколог» Москва 2007г.

Подробнее

ОЛИМПИАДА ПО МАТЕМАТИКЕ 4 класс

Шифр участника Муниципальный этап 2016/2017 учебный год ОЛИМПИАДА ПО МАТЕМАТИКЕ 4 класс Задание 1. Пятьсот пятьдесят пять девяток сложили, а сумму разбили на пятерки. Сколько получилось пятерок? Задание

Подробнее

Задания для учащихся 6 класса

Задания для учащихся 6 класса 1. Саша тренировал глазомер, оценивая длину некоторой дистанции вдоль шоссе. При первой попытке он оценил её в 120 шагов. Оказалось, что за 120 шагов он не дошёл до её конца

Подробнее

«Думай, считай, отгадывай!»

«Думай, считай, отгадывай!» Цели игры: — создать условия для развития математического мышления, быстроты реакции; — содействовать формированию интереса к математике; — обобщать ранее изученный материал.

Подробнее

Электронная школа Знаника

0 Электронная школа Знаника Задача 1 (2 балла) Разбор заданий тестовой части. 4-5 класс 1 Разбор задач тестовой части заданий. 4-5 классы Во сколько раз минутная стрелка двигается быстрее часовой? А. В

Подробнее

Затруднительные положения

Затруднительные положения Задача 1. Как разделить сотню орехов между 25 людьми, чтобы никому не досталось четное число орехов? Решение. Задача неразрешима. Если бы число 100 можно было разбить на 25 нечетных

Подробнее

Развивающие упражнения на память

Развивающие упражнения на память 1. Упражнение на развитие объема кратковременной слуховой памяти: «Каскад слов» Заранее подготовить список из 8-10 слов, которые хорошо знакомы ребенку. Расположите их

Подробнее

Решения для 4 5 класса

1 1. Первая часть Задача 1: Решения для 4 5 класса Сколькими способами можно поставить двух королей одного цвета на доске 3х3 так, чтобы они не били друг друга? А.

4 Б. 8 В. 12 Г. 16 В центре доски король

Подробнее

Error! Reference source not found. 1

Error! Reference source not found. 1 2 Электронная физико-техническая школа Решебник для 4-5 класса 1 Первая часть задания Задача 1: Азартный мальчик Вова купил 12 лотерейных билетов, заплатив за каждый

Подробнее

Ошибка! Источник ссылки не найден. 1

Ошибка! Источник ссылки не найден. 1 2 Электронная школа Знаника Решебник для 4 5 класса Первая часть Задача 1 После ремонта часы Оксаны шли правильно, но рассеянный мастер установил в часах две стрелки

Подробнее

Задания для учащихся 4 5 классов

Задания для учащихся 4 5 классов 1 часть 1. После ремонта часы Оксаны шли правильно, но рассеянный мастер установил в часах две стрелки одинаковой длины. Сколько раз в течение суток Оксана могла видеть

Подробнее

Издательство АСТ Москва

Издательство АСТ Москва Работа. За день обезьянка съедает кг бананов. За сколько дней обезьян съедят кг бананов?. В пакете 9 ирисок и карамелек. Какое наименьшее количество конфет нужно взять не глядя,

Подробнее

«Клуб веселых математиков»

МБОУ «Большебыковская СОШ» «Клуб веселых математиков» Математический турнир Цель: развивать интерес детей к математике, развивать математические способности младших школьников, формировать умение использовать

Подробнее

Межшкольная олимпиада. 4 класс

Межшкольная олимпиада 30.01.16 4 класс 1. Соедините пять звеньев цепи в одну цепь при помощи только шести операций (операции состоят из расковывания и заковывания колец) 2. Скорый поезд вышел из Москвы

Подробнее

Разбор задач четвертой части заданий

Разбор задач четвертой части заданий 1 2 Электронная школа Знаника Разбор задач четвертой части заданий 7 класс 6 7 8 9 10 А Б Б А В Задача 6 Сравните длину пути l, преодолеваемого концом минутной стрелкой

Подробнее

Решения, указания, ответы для 3 класса

ЦЕНТР «ИНТЕЛЛЕКТУАЛЬНЫЙ ДАГЕСТАН» www.int-dag.ru E-mail: [email protected] Региональное отделение общероссийской общественной организации «Малая академия наук «Интеллект будущего» I межрегиональная олимпиада

Подробнее

М А Т Е М А Т И К А В Ш К О Л Е

Тесты по математике по учебнику М. И. Моро для 1 класса. 1 и 2 четверти Тесты по математике для 1 класса, 1 четверть Тест 1 Вариант I 1. На рисунке изображены круги и квадраты. Закрась синим цветом фигуры,

Подробнее

Ошибка! Источник ссылки не найден. 1

Ошибка! Источник ссылки не найден. 1 2 Электронная школа Знаника Решения для 4-5 классов 1 Первая часть заданий 1 2 3 4 5 6 7 8 9 10 11 12 13 14 15 В Б В В Б Б А В Б Г А В А В Б Задача 1 В классе в течение

Подробнее

Ошибка! Источник ссылки не найден. 1

Ошибка! Источник ссылки не найден. 1 2 Электронная школа Знаника Решебник для 4 5 класса Вариант 1. Ответы записывай в специальный бланк ответов. Выполняя тестовые задания, выбирай правильный ответ из

Подробнее

1.Знакомьтесь- маленькое слово «НА».

1.Знакомьтесь- маленькое слово «НА». Запомни- маленькое слово «НА» обозначает, что один предмет находится сверху и соприкасается с другим предметом. Посмотри как выглядит схема этого маленького слова:

Подробнее

1 серия (сентябрь) учебного года класс

1 серия (сентябрь) учебного года 2014-2015 5 класс Напишите наименьшее число, составленное из десяти различных цифр, делящееся на 2. Напишите наибольшее восьмизначное число, делящееся на 5. Напишите число,

Подробнее

Конспект. для детей старшей группы 5-6 лет

Муниципальное дошкольное образовательное учреждение детский сад 41 р.п. Петровское Конспект образовательной деятельности по познавательному развитию на тему: «Полет с Незнайкой на математическую планету»

Подробнее

РАЗВИВАЮЩАЯ ИГРА «СПИЧКИ-ГОЛОВОЛОМКИ»

РАЗВИВАЮЩАЯ ИГРА «СПИЧКИ-ГОЛОВОЛОМКИ» -50 ЗАДАЧ-ГОЛОВОЛОМОК ДЛЯ ИГРЫ НЕ ТРЕБУЕТСЯ СПЕЦИАЛЬНОЙ ДОСКИ, ВЫ МОЖЕТЕ РАЗМЕСТИТЬ СПИЧКИ НА ЛЮБОЙ ПЛОСКОЙ ПОВЕРХНОСТИ. ИНСТРУКЦИЯ ПО ЭКСПЛУАТАЦИИ. ЗАДАЧА 1 ПЕРЕСТАВЬТЕ

Подробнее

0:1 Маша написала фразу: «Я хочу участвовать в Лиге Открытий», посчитала количество букв в каждом слове и перемножила полученные числа. Какой результат должен получиться? 0:0 Сколько всего квадратов изображено

Подробнее

Урок по математике «Решение логических задач» 1 класс

Вводное задание

Решить примеры и буквы подставить под соответствующие цифры

1+8-7=2 Д 15-6+1=10 З (5-4)+3=4А

14-(9+1)=4 А 6+3=9 Ч 10+9-5=14 И (5мин)

-Вышло слово ЗАДАЧИ.

-Ребят какие задачи мы будем решать сегодня на уроке? Мы будем сегодня учиться решать логические задачи.

Но прежде чем приступить к решениям задач вспомним что такое логические задачи

давайте посмотрим какой у нас сегодня необычный урок. Наш урок сегодня будет проходить в кафе. Что нужно делать в кафе?

Какие правила поведения мы знаем в кафе?

-Правила общения

-Правильно пользоваться столовыми приборами и конечно же уметь заказывать еду по меню.

Посмотрите у вас на партах лежат интересные меню.

Что же нам сегодня приготовили на обед это кафе.

Прежде чем заказать еду мы должны что-нибудь по пить

№1 Задание Напитки

Сок яблочный и сок вишневый

Для того чтобы нам принесли сок мы должны поделиться на группы.

1ряд и 2-ряд

Посмотрели у вас на руках геометрическое задание.

1.Нужно посчитать сколько фигур на данном задании?

2.Дать определение данной геометрической фигуре.

Выйти к доске объяснить как они посчитали эти фигуры

Готовая команда показывает свою готовность и один человек выступает.

По исполнению этих заданий мы приклеиваем на меню звездочки. Обратите их внимание, что на рисунках происходит наложение фигур одна на другую и в результате образуются новые фигуры.

5 мин

Следущее задание салат

1 Овощной

2 Фруктовый

Реши. Предложить парами данное задание выполнить практически. Для этого раздать каждой паре по 6 бумажных груш и 2 яблока. Попроситьвзять из массы фруктов четыре фрукта разными способами и варианты фиксировать на листе. Предложить каждой группе назвать количество вариан­тов, а затем озвучить по одному варианту.

По выполнению задач раздать смайлы 3 мин

Горячее

Задание со спичками.Предложить учащимся обсудить данное задание в  парах или индивидуально. Сообщить, что верный ответ нужно написать на листе бумаги.

Выйти показать ответ.

Ответ 4-3=1

9-7=2

1+4=5

9-9=0

5+1=6

5+5=10

8+0=8

5 мин

Ну какое же кафе без музыки.

Физминутка.

Десерт

Выполнить смотрим на учебник или на презентации

У каждого индивидуально спросить их ответы 3 мин

По выполнению смайлик

Дополнительные задания

Развитие языковых компетенций Логические задания предполагают развитие

умения рассуждать, делать выводы. Добивайтесь, чтобы первоклассники давали полные ответы, объясняли свое решение. Предложите им задания­шу тки, попросите их рассуждать, прежде чем отвечать.

Задания­шутки:

• На грядке сидят 5 сорок. К ним прилетели

еще 3. Кот подкрался и схватил одну сороку.

Сколько сорок на грядке? (Нисколько, так как

остальные сороки улетели.)

• На груше росло 10 груш, а на иве на 2 меньше.

Сколько груш росло на иве? (Нисколько, так как

на иве груши не растут).

• На ветке сидело 4 вороны. Охотник выстрелил и

убил одну. Сколько ворон осталось? (Нисколько,

так как все остальные улетели).

Сделайте вывод вместе с  учащимися о том, что

при решении логических задач не следует сразу

приступать к вычислениям. Вначале необходимо

представить себе условие задачи, а затем обдумат

Самые интересные задачи на логику

Дав­но не было зада­чек! Вот под­бор­ка логи­че­ских задач, кото­рые ста­вят в тупик боль­шин­ство взрос­лых, обра­зо­ван­ных людей.

Задача про хитрого электрика

Одна­жды в сек­рет­ном каби­не­те что-то слу­чи­лось с про­вод­кой, и охра­на вызва­ла элек­три­ка, что­бы он всё почи­нил. Ему ска­за­ли, что три выклю­ча­те­ля нахо­дят­ся сна­ру­жи, а три лам­поч­ки — внут­ри. Лам­поч­ки сей­час не горят. Каж­дый выклю­ча­тель отве­ча­ет толь­ко за свою лам­поч­ку, но точ­ной схе­мы не зна­ет никто.

Элек­три­ку ска­за­ли как угод­но щёл­кать выклю­ча­те­ля­ми сна­ру­жи, но внутрь зай­ти раз­ре­ши­ли толь­ко один раз. Внут­ри с лам­поч­ка­ми тоже мож­но было делать что угод­но, но по сооб­ра­же­ни­ям сек­рет­но­сти воз­вра­щать­ся к выклю­ча­те­лям уже нель­зя. Элек­трик ухмыль­нул­ся, пощёл­кал выклю­ча­те­ля­ми, зашёл в ком­на­ту и сра­зу ска­зал, какой выклю­ча­тель отве­ча­ет за каж­дую лам­поч­ку. Как он это сделал?

Если решать зада­чу в лоб, то сра­зу напра­ши­ва­ет­ся такое реше­ние: вклю­чить одну лам­пу и выклю­чить дру­гую. В ито­ге, когда мы зай­дём в ком­на­ту, одна будет гореть, а дру­гая — нет, и мы пой­мём, какой выклю­ча­тель за что отвечает.

Но что делать с тре­тьей лам­пой? Если мы вклю­чим и её, то как отли­чим от такой же пер­вой? А если выклю­чим, то как отли­чим от нера­бо­та­ю­щей вто­рой? Нуж­но научить­ся раз­ли­чать две оди­на­ко­вые рабо­та­ю­щие или нера­бо­та­ю­щие лампы.

Самый про­стой спо­соб это сде­лать — раз­де­лить сами лам­пы допол­ни­тель­но на тёп­лые и холод­ные. Лам­па ста­но­вит­ся тёп­лой, когда пора­бо­та­ет, и даже если её выклю­чить, она всё рав­но какое-то вре­мя оста­нет­ся тёплой.

По усло­вию мы зна­ем, что все три лам­пы выклю­че­ны. Но вдруг они недав­но вклю­ча­лись и ещё не успе­ли остыть? Зна­чит, пер­вое, что мы дела­ем, — ждём неко­то­рое вре­мя, что­бы все лам­пы остыли.

Теперь щёл­ка­ем любым выклю­ча­те­лем и нагре­ва­ем одну лам­пу. После того, как она пора­бо­та­ла доста­точ­но вре­ме­ни, что­бы нагреть­ся, мы её выклю­ча­ем. Полу­ча­ет­ся, что у нас все три лам­пы выклю­че­ны, но две из них холод­ные, а одна — тёплая.

Затем, что­бы раз­ли­чить две холод­ные лам­пы, щёл­ка­ем любым дру­гим выклю­ча­те­лем и захо­дим в ком­на­ту. В ито­ге мы увидим:

  • одну рабо­та­ю­щую лам­поч­ку, кото­рую мы вклю­чи­ли толь­ко что;
  • одну нера­бо­та­ю­щую, но тёп­лую лам­поч­ку, кото­рую мы нагре­ли до этого;
  • и одну нера­бо­та­ю­щую и холод­ную лам­поч­ку, выклю­ча­тель от кото­рой мы ни разу не трогали.

Теп­ло и логика!

Новые приключения хитрого электрика

Один про­вай­дер решил про­ве­сти интер­нет через реку — от лево­го бере­га до пра­во­го. Для это­го он под водой про­ло­жил 49 про­во­дов, по кото­рым пере­да­ют­ся сиг­на­лы и элек­три­че­ский ток.

Все про­во­да ока­за­лись оди­на­ко­во­го цве­та, а под­ряд­чик забыл про­мар­ки­ро­вать их, что­бы понять, где какие кон­цы про­во­дов на обо­их берегах.

Что­бы выяс­нить, где что, позва­ли элек­три­ка и ска­за­ли ему под­пи­сать все про­во­да чис­ла­ми от 1 до 49 с каж­дой сто­ро­ны. Его зада­ча — про­ну­ме­ро­вать про­во­да на левом бере­гу и на пра­вом, разу­ме­ет­ся, что­бы чис­ла совпали.

Ему предо­ста­ви­ли катер, кото­рый может возить его сколь­ко угод­но раз с одно­го бере­га на дру­гой, линию с током на исход­ном бере­гу и муль­ти­метр, кото­рый пока­зы­ва­ет напря­же­ние в проводе.

Все дума­ли, что элек­трик пере­се­чёт реку как мини­мум 49 раз, но ему хва­ти­ло все­го двух раз — туда и обрат­но. Потом он про­сто сидел на бере­гу и задум­чи­во смот­рел на воду. Как ему это удалось?

На исход­ном бере­гу элек­трик пода­ёт напря­же­ние на любой про­вод и поме­ча­ет его как № 1. Все осталь­ные 48 он попар­но соеди­ня­ет меж­ду собой, что­бы на этой сто­роне полу­чил­ся один про­вод под напря­же­ни­ем и 24 пары. Как он это дела­ет — вооб­ще не важ­но, поря­док пар сей­час роли не игра­ет. После это­го элек­трик отправ­ля­ет­ся на пра­вый берег (пер­вая поездка).

При­плыв на место, он нахо­дит про­вод под напря­же­ни­ем с помо­щью тесте­ра — это про­вод № 1, он его так и поме­ча­ет. А даль­ше начи­на­ет­ся элек­три­че­ская магия.

Элек­трик берёт про­вод № 1 под напря­же­ни­ем, соеди­ня­ет его с любым дру­гим про­во­дом и под­пи­сы­ва­ет его как № 2. Но мы пом­ним, что на левом бере­гу все про­во­да соеди­не­ны попар­но, зна­чит, про­вод № 2 с той сто­ро­ны тоже с чем-то соеди­нён, а зна­чит, ток вер­нёт­ся обрат­но и появит­ся в новом про­во­де, кото­рый элек­трик под­пи­шет как № 3.

Даль­ше всё то же самое: он берёт про­вод с током № 3, соеди­ня­ет его с любым остав­шим­ся про­во­дом и под­пи­сы­ва­ет новый про­вод как № 4. А ещё он пом­нит про пары на том бере­гу, поэто­му ищет про­вод, в кото­ром сно­ва появил­ся ток и под­пи­сы­ва­ет его как № 5. Таким же обра­зом он соеди­ня­ет остав­ши­е­ся про­во­да и нуме­ру­ет все жилы на пра­вой сто­роне от 1 до 49. Сде­лав это, элек­трик воз­вра­ща­ет­ся на левый берег (вто­рая поездка).

Оста­лось самое инте­рес­ное: как на этом бере­гу про­ста­вить те же самые чис­ла на про­во­дах. Элек­трик зна­ет, как выгля­дит про­вод № 1, пото­му что он его под­пи­сал, но не зна­ет, как выгля­дит про­вод № 2.

Но он пом­нит, что про­вод № 1 соеди­нён на том бере­гу с про­во­дом № 2, кото­рый на этом бере­гу соеди­нён с про­во­дом № 3. Зна­чит, зада­ча элек­три­ка в том, что­бы най­ти это соеди­не­ние на левом бере­гу, где он нахо­дит­ся. Для это­го он разъ­еди­ня­ет по оче­ре­ди все соеди­не­ния и смот­рит, про­пал ли ток во всех осталь­ных про­во­дах. Если не про­пал во всех осталь­ных — зна­чит, разъ­еди­нил не ту пару и воз­вра­ща­ет её на место. А если про­пал — зна­чит, элек­трик нашёл соеди­не­ние про­во­дов № 2 и № 3. При этом тот неиз­вест­ный про­вод, кото­рый остал­ся под напря­же­ни­ем, будет про­вод № 2, а тот, с кото­рым он соеди­нял­ся, будет № 3.

После это­го элек­трик соеди­ня­ет под­пи­сан­ную пару обрат­но и начи­на­ет искать сле­ду­ю­щую точ­ку, кото­рая отклю­ча­ет все осталь­ные жилы — это будут про­во­да № 4 и № 5. Дей­ствуя по этой схе­ме, хит­рый элек­трик под­пи­шет все остав­ши­е­ся про­во­да. Про­вай­де­ру оста­нет­ся толь­ко разъ­еди­нить пары на каж­дом берегу.

Как перевезти гопников и философов с одного берега на другой

На одном бере­гу реки нахо­дят­ся шесть чело­век: три гоп­ни­ка и три фило­со­фа. Пока что они ведут непри­нуж­дён­ные бесе­ды об экзи­стен­ци­аль­ном, но все долж­ны будут рано или позд­но ока­зать­ся на дру­гом берегу.

Есть одна лод­ка, в кото­рую могут поме­стить­ся толь­ко два чело­ве­ка, но фило­со­фы управ­лять лод­кой не уме­ют, а гоп­ни­ки уме­ют. Так­же нель­зя остав­лять на одном бере­гу фило­со­фов боль­ше, чем гоп­ни­ков, пото­му что тогда фило­со­фы взо­рвут мозг гоп­ни­кам раз­го­во­ра­ми о при­ро­де вещей. Как пере­пра­вить всех через реку?

Для пер­вой поезд­ки есть пять вариантов: 

  • один гоп­ник — не под­хо­дит, пото­му что на бере­гу фило­со­фов ста­но­вит­ся боль­ше и они взо­рвут мозг;
  • два гоп­ни­ка — не под­хо­дит по той же причине;
  • один или два фило­со­фа — тоже нет, пото­му что они не уме­ют управ­лять лодкой;
  • фило­соф и гоп­ник — един­ствен­ный вари­ант, кото­рый остаётся.

Зна­чит, пер­вым рей­сом пара «философ-гопник» отправ­ля­ет­ся на дру­гой берег:

Теперь лод­ку надо как-то отпра­вить назад. Но так как фило­соф не уме­ет ей управ­лять, то он оста­ёт­ся на бере­гу, а гоп­ник — воз­вра­ща­ет­ся. Фило­со­фы не взры­ва­ют нико­му мозг:

Теперь при­ки­нем вари­ан­ты сле­ду­ю­ще­го рей­са. Мы не можем отпра­вить двух гоп­ни­ков, ина­че фило­со­фы оста­нут­ся в боль­шин­стве, и наста­нет на левом бере­гу пол­ный экзистенциализм.

Поэто­му сно­ва на тот берег уплы­ва­ют фило­соф с гоп­ни­ком. При­чём гоп­ник выса­жи­ва­ет фило­со­фа, но сам из лод­ки не выле­за­ет — если так не сде­лать, то он оста­нет­ся с дву­мя фило­со­фа­ми на том бере­гу и они увле­кут раз­го­во­ра­ми об иде­ях вещей:

Таким обра­зом, у нас на том бере­гу сидят два фило­со­фа, а на этом — один фило­соф и три гоп­ни­ка, на кото­рых он вряд ли смо­жет воз­дей­ство­вать силой дискурса:

Теперь нам нуж­но сде­лать выбор, кто поедет на этот раз. Мож­но отпра­вить сно­ва фило­со­фа и гоп­ни­ка, но тогда на том бере­гу ока­жут­ся три фило­со­фа. И без­опас­но пере­вез­ти осталь­ных гоп­ни­ков пооди­ноч­ке уже не полу­чит­ся — фило­со­фы все­гда будут в большинстве.

Зна­чит, оста­ёт­ся толь­ко один вари­ант: отпра­вить в путь двух гоп­ни­ков. В ито­ге на том бере­гу всех будет поров­ну и всё прой­дёт спокойно:

Но лод­ку надо как-то отпра­вить на дру­гой берег. Нель­зя раз­ме­стить на ней одно­го гоп­ни­ка, пото­му что вто­рой оста­нет­ся в мень­шин­стве сре­ди фило­со­фов. Двум гоп­ни­кам ехать обрат­но тоже не вари­ант, пото­му что они толь­ко что прибыли.

Поэто­му назад отправ­ля­ют­ся фило­соф и гопник:

Теперь един­ствен­ный без­опас­ный вари­ант — отпра­вить на тот берег двух гопников:

Назад отпра­вим одно­го гоп­ни­ка. Что­бы не выхо­дить из лод­ки, он позо­вёт в неё фило­со­фа (напри­мер, фра­зой «Что вы дума­е­те о солип­сиз­ме?») и вер­нёт­ся с ним обрат­но на тот берег:

Точ­но так же заби­ра­ем остав­ше­го­ся философа:

И в ито­ге вся ком­па­ния ока­зы­ва­ет­ся на том бере­гу, без­дон­ное небо — над голо­вой, а нрав­ствен­ный закон — внутри:

Как рассадить интровертов в баре

А вот задач­ка на струк­ту­ры дан­ных, сор­ти­ров­ку и алго­рит­ми­ку, кото­рая воз­мож­на толь­ко в нашей стране.

В Петер­бур­ге на ули­це Рубин­штей­на есть один бар, в кото­рый ходят лишь необ­щи­тель­ные люди, назо­вём их интро­вер­та­ми. (На самом деле интро­вер­ты общи­тель­ные, необ­щи­тель­ность — это миф. Но это задач­ка, поэто­му упростим.)

Интро­вер­ты садят­ся вдоль бар­ной стой­ки, где есть 25 мест. Когда вхо­дит новый посе­ти­тель, он все­гда садит­ся у стой­ки как мож­но даль­ше от осталь­ных гостей. Никто не садит­ся на сосед­нее место рядом с дру­гим интро­вер­том: если кто-то вхо­дит и видит, что сво­бод­ных мест мало и надо сесть рядом с кем-то, то он уходит.

Бар­мен хочет полу­чить как мож­но боль­ше кли­ен­тов. У него есть пра­во поса­дить само­го пер­во­го посе­ти­те­ля на любое место у стой­ки. Куда выгод­нее поса­дить пер­во­го интро­вер­та с точ­ки зре­ния бармена?

Для нача­ла най­дём иде­аль­ный вари­ант, кото­рый устро­ил бы бар­ме­на. Для это­го нари­су­ем 25 квад­ра­тов в ряд и закра­сим те, на кото­рых кто-то сидит. Помни­те, что ни один интро­верт по зада­че не сядет на сосед­нее место к другому.

Полу­ча­ет­ся, что это самая плот­ная рас­сад­ка, кото­рая воз­мож­на в этом баре. Так у стой­ки сидят 13 чело­век. Оста­лось толь­ко най­ти место для само­го пер­во­го посетителя.

Для нача­ла попро­бу­ем решить эту зада­чу в лоб и поса­дим пер­во­го посе­ти­те­ля на пер­вый стул:

Теперь вто­рой посе­ти­тель дол­жен сесть на сво­бод­ное место как мож­но даль­ше от него, то есть занять стул № 25:

Тре­тье­му доста­ёт­ся стул № 13, так как он ров­но посе­ре­дине меж­ду эти­ми двумя:

Два сле­ду­ю­щих зай­мут сво­бод­ные места точ­но посе­ре­дине меж­ду цен­траль­ным и боковыми:

И вот тут наста­ёт момент исти­ны: четы­ре сле­ду­ю­щих посе­ти­те­ля тоже сядут точ­но посе­ре­дине меж­ду заня­ты­ми места­ми. Это зна­чит, что меж­ду каж­дым будет по 2 пустых места:

В ито­ге у нас заня­то все­го 9 мест, но сесть боль­ше нику­да нель­зя: у каж­до­го сво­бод­но­го сту­ла есть как мини­мум один заня­тый сосед. Зна­чит, этот вари­ант не под­хо­дит. Нужен другой.

Что­бы прий­ти к пра­виль­но­му отве­ту, попро­бу­ем решать зада­чу с конца.

Вспом­ним иде­аль­ную рассадку:

Здесь сидит мак­си­маль­ное коли­че­ство гостей — 13, и меж­ду каж­дым из них есть сво­бод­ное место. Отмо­та­ем на шаг назад и посмот­рим, как мог­ли бы сидеть интро­вер­ты, что­бы новые гости сели точ­но меж­ду ними:

В этом слу­чае 6 новых гостей садят­ся точ­но посе­ре­дине меж­ду заня­ты­ми сту­лья­ми и иде­аль­но запол­ня­ют все места.

Теперь сде­ла­ем ещё шаг назад и посмот­рим, как долж­ны сидеть гости, что­бы новые кли­ен­ты сели на нуж­ные стулья:

Полу­ча­ет­ся, что если мы поса­дим пер­вых четы­рёх гостей так, как на рисун­ке выше, то даль­ше всё будет хоро­шо. Сде­ла­ем ещё шаг назад, что­бы понять, как они смог­ли так сесть:

Из рисун­ка вид­но, что два новых посе­ти­те­ля долж­ны сесть как мож­но даль­ше от заня­тых мест. Для это­го один садит­ся ров­но посе­ре­дине меж­ду дву­мя заня­ты­ми, а вто­рой — с само­го края, на пер­вое место. Таким обра­зом, меж­ду все­ми ними будет мак­си­маль­но воз­мож­ное рас­сто­я­ние. Оста­лось понять, как сели эти пер­вые два интроверта.

Если бы пер­вый гость сел с краю на стул № 25, вто­ро­му бы при­шлось сесть с про­ти­во­по­лож­но­го края на стул № 1 (мы это разо­бра­ли в самом нача­ле, в непра­виль­ном вари­ан­те). Зна­чит, пер­вый гость сел на стул № 9, а вто­ро­му при­шлось сесть мак­си­маль­но дале­ко от него — на самый послед­ний стул:

Полу­ча­ет­ся, само­го пер­во­го гостя бар­мен дол­жен поса­дить на стул № 9.

Как так вышло? Про­сто посчи­та­ли от обрат­но­го. Про­грам­ми­сты назы­ва­ют это Test-First Development, хех. 

Логическая задача про лифт

Одна­жды в 20-этажном доме вандалы-математики раз­би­ли почти все кноп­ки в лиф­те, сохра­нив толь­ко две. От корот­ко­го замы­ка­ния послед­ние ста­ли рабо­тать так: одна под­ни­ма­ет лифт на 13 эта­жей, а вто­рая опус­ка­ет на 8.

Как жиль­цам попасть с 13-го эта­жа на 8-й?

В этой зада­че есть момент из реаль­ной жиз­ни, кото­рый суще­ствен­но упро­ща­ет реше­ние. Но нач­нём с клас­си­че­ско­го ответа.

Суть в том, что лифт не может выез­жать за гра­ни­цы эта­жей. То есть если на 13 эта­же мы нажмём кноп­ку «вверх», кото­рая долж­на под­нять лифт на 13 эта­жей, то он нику­да не поедет, пото­му что 13 + 13 = 26, а в доме столь­ко эта­жей нет. Зна­чит, един­ствен­ное, что нам оста­ёт­ся на пер­вом шаге — нажать «вниз»:

Вниз → 5 (13 — 8).

Здесь 5 — это номер эта­жа, на кото­рый при­е­хал лифт, а циф­ры в скоб­ках пока­зы­ва­ют начальный.

С 5 эта­жа мы можем уехать толь­ко вверх. Полу­ча­ет­ся, что каж­дый раз у нас есть толь­ко один вари­ант, на какую кноп­ку нажи­мать. Давай­те попро­бу­ем при­ме­нить этот прин­цип и посмот­реть, что получится:

Вниз → 5 (13 — 8).

Вверх → 18 (5 + 13).

Вниз → 10 (18 — 8).

Вниз → 2 (10 — 8).

Вверх → 15 (2 + 13).

Вниз → 7 (15 — 8).

Вверх → 20 (7 + 13).

Вниз → 12 (20 — 8).

Вниз → 4 (12 — 8).

Вверх → 17 (4 + 13).

Вниз → 9 (17 — 8).

Вниз → 1 (9 — 8).

Вверх → 14 (1 + 13).

Вниз → 6 (14 — 8).

Вверх → 19 (6 + 13).

Вниз → 11 (19 — 8).

Вниз → 3 (11 — 8).

Вверх → 16 (3 + 13).

Вниз → 8 (16 — 8).

В ито­ге за 19 поез­док мы добра­лись до нуж­но­го эта­жа. Самое инте­рес­ное, что по этим пра­ви­лам лифт даль­ше нику­да поехать не может: 8 + 13 = 21, а 8 — 8 = 0, что выхо­дит за гра­ни­цы эта­жей. При­дёт­ся всё-таки вызы­вать масте­ра и делать ремонт.

Но есть и вто­рое реше­ние. Чаще все­го в жиз­ни быва­ет так: как толь­ко лифт доез­жа­ет до само­го верх­не­го или ниж­не­го эта­жа, он оста­нав­ли­ва­ет­ся, неза­ви­си­мо от того, сколь­ко ещё ему оста­ва­лось про­ехать. Это логич­но: дошли до гра­нич­ных зна­че­ний и оста­но­ви­лись. Вос­поль­зу­ем­ся этим и попро­бу­ем решить нашу зада­чу быстрее:

Вниз→ 5 (13 — 8).

Вниз → 1 (5 — 8) → дое­ха­ли до пер­во­го эта­жа и остановились.

А как добрать­ся с 1 эта­жа на 13 мы уже зна­ем из про­шло­го решения:

Вверх → 14 (1 + 13).

Вниз → 6 (14 — 8).

Вверх → 19 (6 + 13).

Вниз → 11 (19 — 8).

Вниз → 3 (11 — 8).

Вверх → 16 (3 + 13).

Вниз → 8 (16 — 8).

Ито­го 9 поез­док. В два раза мень­ше, чем пер­вым способом!

Граж­дане, бере­ги­те лифт!

Находчивый инженер в кафе

В кафе поста­ви­ли 3 раз­ных авто­ма­та, кото­рые нали­ва­ют напит­ки. В пер­вом – кофе, во вто­ром – чай, а в тре­тий выда­ёт слу­чай­ным обра­зом то кофе, то чай (пото­му что в жиз­ни все­гда долж­но быть место экс­пе­ри­мен­ту). Для каж­до­го из авто­ма­тов нуж­на 1 моне­та, что­бы полу­чить напиток.

На заво­де пере­пу­та­ли мар­ки­ров­ку авто­ма­тов, поэто­му на каж­дом из них ока­за­лась непра­виль­ная наклей­ка. Сколь­ко монет пона­до­бит­ся наход­чи­во­му инже­не­ру, что­бы понять, где какой автомат?

Несмот­ря на то что зада­ча кажет­ся запу­тан­ной, у неё доволь­но изящ­ное реше­ние. Сле­ди­те за рука­ми наход­чи­во­го инженера.

Кида­ем моне­ту в авто­мат с наклей­кой «Чай-кофе». Мы зна­ем, что на нём непра­виль­ная наклей­ка, как и на всех, поэто­му пра­виль­ная будет либо «Чай», либо «Кофе». Теперь смот­рим, что нам выдаст этот автомат.

Напри­мер, он выдал чай. Зна­чит, пра­виль­ная наклей­ка для это­го авто­ма­та — «Чай». Теперь нам нуж­но най­ти кофей­ный авто­мат сре­ди двух оставшихся.

Мы пом­ним, что все наклей­ки пере­пу­та­ны, поэто­му там, где будет напи­са­но «Кофе», на самом деле не кофей­ный авто­мат. Чай тоже уже занят. Поэто­му под над­пи­сью «Кофе» скры­ва­ет­ся авто­мат, кото­рый выда­ёт и кофе, и чай.

Зна­чит, с наклей­кой «Чай» будет авто­мат, кото­рый выда­ёт кофе.

О чудо! Что­бы разо­брать­ся с наклей­ка­ми, доста­точ­но все­го одной монеты!

Как успеть на презентацию

Илон Маск, Билл Гейтс, Тим Кук и Марк Цукер­берг хотят пер­вы­ми попасть на пре­зен­та­цию Xiaomi, поэто­му реши­ли вый­ти ночью, что­бы к утру быть уже на месте. Кру­гом тем­но­та, без фона­ри­ка нико­му идти нель­зя, но он один на всех. Пре­зен­та­ция — на дру­гом бере­гу вели­кой реки Янц­зы. Мост через реку хлип­кий и может выдер­жать одно­вре­мен­но мак­си­мум дво­их. Как всем пере­брать­ся на дру­гой берег как мож­но скорее?

Ско­рость пере­хо­да моста у каж­до­го своя: про­вор­ный Илон Маск пере­хо­дит его за 1 мину­ту, бод­ря­щий­ся Билл Гейтс — за 2, спо­кой­ный Тим Кук — за 5. Марк Цукер­берг после слу­ша­ний в Кон­грес­се быст­ро ходить не может, поэто­му тра­тит на мост 10 минут. Когда мост пере­хо­дят два чело­ве­ка, их ско­рость рав­на ско­ро­сти само­го мед­лен­но­го из пары.

Зада­ча — пере­ве­сти геро­ев на дру­гой берег как мож­но ско­рее, ведь места в оче­ре­ди у конгресс-центра уже зани­ма­ют мест­ные жители.

Самая ско­рост­ная пара у нас — Маск и Гейтс, поэто­му они с фона­ри­ком пере­хо­дят на дру­гой берег за 2 мину­ты (ско­рость Гейтса):

Илон Маск (1) и Билл Гейтс (2) → пере­шли на тот берег за 2 минуты.

Отправ­ля­ем с фона­рём назад само­го быст­ро­го из них:

Илон Маск (1) → вер­нул­ся обрат­но с фона­рём за 1 минуту.

Теперь нуж­но решить, какая пара пой­дёт сле­ду­ю­щей. Так как нам в любом слу­чае нуж­но отправ­лять Цукер­бер­га на тот берег, то это гаран­ти­ро­ван­но зай­мёт дол­гих 10 минут. Что­бы исполь­зо­вать это вре­мя опти­маль­но, отпра­вим с ним Тима Кука, кото­рый тоже не самый быст­рый из всех:

Тим Кук (5) и Марк Цукер­берг (10) → пере­шли на тот берег за 10 минут.

Оста­лось забрать Ило­на Мас­ка с того бере­га, зна­чит посы­ла­ем за ним само­го быст­ро­го из доступ­ных — Бил­ла Гейтса:

Билл Гейтс (2) → вер­нул­ся обрат­но с фона­рём за 2 минуты.

И они вдво­ём с Мас­ком отправ­ля­ют­ся на тот берег:

Илон Маск (1) и Билл Гейтс (2) → пере­шли на тот берег за 2 минуты.

Скла­ды­ва­ем все мину­ты на мосту: 2 + 1 + 10 + 2 + 2 = 17 минут. Зна­чит, все­го 17 минут им потре­бу­ет­ся, что­бы перей­ти вели­кую реку Янц­зы и занять места в зале рань­ше всех.

Находчивый альпинист

Один аль­пи­нист неудач­но спу­стил­ся с горы и насту­пил сра­зу на двух змей — коб­ру и гадю­ку. Одна из них его уку­си­ла, какая — неиз­вест­но. У него были с собой про­ти­во­ядия, по две таб­лет­ки каж­до­го вида: про­тив коб­ры и про­тив гадю­ки. Одну таб­лет­ку нуж­но при­нять сра­зу после уку­са, а дру­гую — на сле­ду­ю­щий день.

Аль­пи­нист вытрях­нул из упа­ков­ки на ладонь одну таб­лет­ку от коб­ры (K), стал вытря­хи­вать таб­лет­ку от гадю­ки (Г), но рука дрог­ну­ла и из упа­ков­ки Г выпа­ли обе таб­лет­ки. Теперь у него в руке три абсо­лют­но оди­на­ко­вые таб­лет­ки: одна K, две Г. А ему нуж­но немед­лен­но при­нять одну K и одну Г, оста­вив по вто­рой таб­лет­ке каж­до­го про­ти­во­ядия на зав­тра. Что ему делать?

Если таб­лет­ки никак нель­зя отли­чить друг от дру­га, зна­чит, надо при­ду­мать такое реше­ние, кото­рое не потре­бу­ет ана­ли­за всех таблеток.

Зада­ча аль­пи­ни­ста — при­нять одну таб­лет­ку от уку­са коб­ры и одну от уку­са гадю­ки. На ладо­ни лежат три таб­лет­ки, и если мы возь­мём любые две, то есть веро­ят­ность, что нам попа­дут­ся две таб­лет­ки от гадю­ки и тогда про­ти­во­ядие от коб­ры не сра­бо­та­ет (аль­пи­нист же не зна­ет, какая имен­но змея его уку­си­ла). Зна­чит, нам такой вари­ант не подходит.

Что­бы сего­дня и зав­тра при­нять оди­на­ко­вые пор­ции, аль­пи­ни­сту нуж­но к этим трём таб­лет­кам доба­вить чет­вёр­тую, раз­ло­мать их все попо­лам и раз­не­сти эти поло­вин­ки по двум раз­ным куч­кам. Смысл в том, что­бы в каж­дой куч­ке лежа­ло по одной поло­вин­ке от каж­дой таб­лет­ки. Тогда в обе­их будет по две поло­вин­ки таб­лет­ки от коб­ры и по две поло­вин­ки таб­лет­ки от гадю­ки, а две поло­ви­ны дают как раз целую таблетку.

Полу­ча­ет­ся, что ему сего­дня и зав­тра нуж­но съесть по 4 поло­вин­ки, по одной от каж­дой таблетки. 

как провести День математика / Новости города / Сайт Москвы

Пройти математический квест, узнать историю предмета и научиться решать самые сложные олимпиадные задачи — «Московская электронная школа» («МЭШ») предлагает учащимся провести День математика с пользой.

Математическая логика

Потренироваться в решении шуточных заданий можно с помощью специального приложения «Логические задачи». Ребятам предлагают ответить на 15 вопросов, где точно понадобится включить логику.

Математика в творчестве

Вспомнить пословицы и поговорки с упоминанием чисел можно с помощью приложения «Числа в пословицах и поговорках». Школьники должны вставить пропущенные числа в известные выражения. Разработчики подготовили 20 заданий.

Задачи международного уровня

Международное исследование качества математического и естественно-научного образования TIMSS (Trends in Mathematics and Science Study) оценивает общеобразовательную подготовку учащихся четвертых и восьмых классов по математике и естественно-научным предметам, а также подготовку учащихся 11-х классов по углубленным курсам математики и физики. Познакомиться с типами и формами заданий, с помощью которых оценивается подготовка выпускников начальной школы, поможет приложение «Задания в формате TIMSS».

Квест

Каждому образованному человеку знаком такой раздел математики, как арифметика. Но не каждый может сказать, кто внес неоценимый вклад в развитие математического просвещения России. Узнать фамилию ученого, который написал первый учебник по математике в нашей стране, поможет специальный квест.

Финансовые задачи

Тест «Финансовые задачи в истории, литературе и математике» поможет узнать, как связана точная наука с другими областями нашей жизни. Пользователи приложения предстоит ответить на 12 вопросов. В каждом можно выбрать один ответ из трех. Пройдет тест тот, кто знает, как называется украшение, сделанное из монет, или какая денежная единица в Таиланде.

День математика отмечается 1 апреля. Праздник носит неформальный характер и пока не имеет официального статуса.

Вопросы-головоломки на собеседованиях с ответами. Часть 1 — Work.ua

Логические задачи кандидатам на вакансию задают, чтобы оценить их аналитические способности. С подобными вопросами может столкнуться каждый. Но даже, если с вами это не случится — изучить их для подготовки будет просто интересно.

Соискателям, столкнувшимся с головоломками в процессе собеседования, не стоит паниковать и теряться. Как правило, в большинстве случаев достаточно показать ход ваших мыслей и пути решения поставленной задачи. Вполне может быть, что у задачи и нет правильного решения или однозначного ответа. Ни в коем случае нельзя говорить «я не знаю» или «затрудняюсь ответить», просто размышляйте вслух.

Чтобы вы знали к чему быть готовыми и не растерялись, Work.ua собрал самые популярные вопросы-головоломки и подготовил ответы и рекомендации к ним. Кто знает, может вам попадутся именно эти задачи.

Все эти головоломки могут вам задать при собеседовании на вакансию программиста php.

Канализационные люки

Это, пожалуй, самый распространенный вопрос среди всех головоломок.

Вопрос: Почему канализационные люки круглые?

Ответ: Есть несколько вариантов. Так как диаметр круга одинаков, как его не крути, то круглый люк не может провалиться в колодец. У квадрата же, например, диагональ больше его сторон, поэтому крышка могла бы упасть. Также можно ответить, что круглые люки проще транспортировать и передвигать.

Лампочки

Вопрос: В закрытой комнате есть 3 лампочки, а в коридоре 3 выключателя. За какое минимальное открытие дверей можно определить какой выключатель к какой лампочке относится?

Ответ: За одно открытие. Включаем одновременно 2 выключателя, через некоторое время выключаем второй. Заходим в комнату: одна из лампочек осталась гореть — это первый выключатель, далее щупаем оставшиеся лампочки — теплая и будет вторым выключателем, а холодная соответственно третьим.

Торт

Вопрос: Как разделить торт на 8 равных частей тремя разрезами?

Ответ: Сперва нужно сделать 2 разреза крест на крест, поделив торт на 4 равных части. А затем разрезать торт горизонтально пополам. Ну и что, что куски стали невысокие, зато у вас 8 равных частей. Еще можно после первых двух разрезов сложить кусочки друг на друга и переполовинить одним разрезом.

Мертвый человек и спичка

Вопрос: Среди поля найден мертвый человек со спичкой в руках, следов нет. От чего он умер и при каких обстоятельствах?

Ответ: Человек умер от падения с самолета, который начал терять высоту, и авария была неизбежна. На всех пассажиров не хватило одного парашюта и они тянули жребий. Ему досталась короткая спичка, и он был вынужден прыгать без парашюта.

Автобус и мячи

Вопрос: Сколько теннисных мячей поместится в автобус?

Ответ: Точного правильного ответа не знает и сам рекрутер, так как не уточняется, что за мячи и автобус — их размеры не известны, и никто досконально это не проверял. Поэтому тут важен ход ваших мыслей, вы можете только предположить. Назовите примерные длину, ширину и высоту автобуса, размеры одного мяча. Посчитайте объем автобуса и мяча — так вы узнаете сколько мячей поместится в пустой автобус. Уменьшите примерно это значение с учетом сидений и других деталей автобуса, сделайте поправку на то, что мячи не квадратные и дайте ответ. В этом случае процесс поиска ответа важнее самого ответа. Варианты вопроса могут быть разные: мячи — футбольные, вместо автобуса — комната и т.п.

Таблетки

Вопрос: Доктор выдал пациенту 4 таблетки двух видов — по 2 таблетки каждого, которые нельзя отличить по внешнему виду. Таблетки надо выпить за два приема: утром по одной таблетке каждого вида и так же вечером. Если нарушить дозировку или не принять таблетки, то пациент умрет. Так вышло, что таблетки перемешались. Как пройти курс лечения и выжить?

Ответ: Конечно, можно сказать, что лучше пойти к врачу и попросить еще, все таки вопрос жизни и смерти. Но это могут быть единственные таблетки на Земле, доктор может исчезнуть при загадочных обстоятельствах и т.д. Так что ответить все же придется. К тому же все достаточно просто: нужно разделить каждую таблетку на 2 части и выпить по половинке каждой таблетки утром и вечером.

В заключение

Помните, что прежде всего такие задачи и головоломки предназначены для того, чтобы проверить поведение кандидата в нестандартных ситуациях, оценить способность к размышлению, творческому и логическому подходу. Увы, нередки случаи, когда интервьюер не может правильно интерпретировать результаты таких вопросов, или вовсе не понимает их предназначения. Но даже в этой ситуации уверенное поведение и стремление прийти к ответу покажут вас с лучшей стороны и увеличат шансы получить должность.


Читайте также: Вопросы-головоломки на собеседованиях с ответами. Часть 2



Чтобы оставить комментарий, нужно войти.

Любимые математические пазлы для детей

В феврале 2014 года я попросил своих подписчиков назвать их любимую математическую головоломку. Результатом стал сборник — список головоломок, подобранный учителями и родителями! Большинство из них требует только четырех основных операций или даже не этого, поэтому они хорошо подходят для детей младшего школьного возраста и выше. Я разделил головоломки на геометрические по своей природе и математические и логические. Сложные головоломки находятся на отдельной странице.Я также перечисляю несколько хороших веб-сайтов с головоломками и книг внизу этой страницы.

Перейти к:

Пазлы с фигурами и пиками
Математические пазлы
Сайты с пазлами

Пазлы с фигурами и палочек

  1. Stardoku

    Это для вас, ребята из детского сада — игра-раскраска судоку! Звезды в одной строке и столбце должны иметь разные цвета.

    игра Стардоку

  2. Поверните рыбу

    Переместите три палки и заставьте рыбу плыть в противоположном направлении.Вы можете увидеть решение и сыграть в интерактивную версию этой головоломки на Puzzles.com (и во многих других!).

  3. Свиньям нужны загоны

    Фермер Блэк разводил свиней и разводил их в модульных загонах. имеет 6 свиней в следующей расстановке:

     + --- + + --- + + --- + + --- + + --- + + --- +
    | | | | | | | | | | | |
    + --- + + --- + + --- + + --- + + --- + + --- +
     
    Ему нужны были стены для другого проекта, поэтому он переставил булавки следующим образом:
    + --- + --- + --- + --- + --- + --- +
    | | | | | | |
    + --- + --- + --- + --- + --- + --- +
    
     
    Затем он обнаружил, что ему нужно еще 7 стеновых панелей, чтобы завершить свой проект.Как он переставил загоны, чтобы в загоне была только 1 свинья?

    Посмотреть решение.


  4. Монетный треугольник

    Переместите всего три монеты и переверните треугольник вверх дном.

    Щелкните здесь, чтобы получить ответ, но не раньше, чем вы задумаетесь об этом!

  5. Точки в квадрате
    Это один из моих любимых! Это простая головоломка, но она прекрасно иллюстрирует идею нестандартного мышления.»

    Соедините все точки четырьмя прямыми линиями, не поднимая карандаша.

    Нажмите здесь, чтобы получить ответ, но сначала подумайте об этом!

  6. Пентагон и звезда

    Сколько там треугольников?

    См. Подсказку и решение на сайте Cool Math 4 Kids.



  7. Ханойская башня

    Ханойская башня — это старая любимая головоломка, в которой вы перемещаете диски из одной башни в другую.Если вы никогда не играли, попробуйте эту онлайн-версию. Вы можете либо сыграть в нее сами, либо позволить компьютеру решить ее.

  8. Деревья рядами

    Садовник посадил 10 деревьев в пять рядов. В каждом ряду было по четыре дерева. Как он это сделал?

    Нажмите здесь, чтобы получить ответ, но не заглядывайте слишком рано — сначала попробуйте честно!

  9. Головоломка со спичками

    Переместите 3 спички, чтобы показать 2 квадрата.

    См. Решение здесь


Математические головоломки

  1. Kenken

    В Kenken вам нужно разместить счетные числа в квадратах, как и в судоку, но, кроме того, вам будут предоставлены коробки с подсказками с математическими операциями. Цифры, которые вы помещаете в каждую ячейку, должны составлять «подсказку» с использованием данной операции.

    Итак, Kenken — отличная игра для отработки четырех операций и логического мышления.Дети младших классов могут использовать сложение / вычитание, а другие могут использовать умножение / деление или все четыре операции.

    Играйте в Kenken онлайн здесь. Вы можете выбрать размер игры, операции и уровень сложности.

  2. Числа в треугольнике

    Поместите числа от 1 до 9 в кружки так, чтобы сумма чисел на каждой стороне треугольника была одинаковой.

    На CoolMath5Kids вы можете просмотреть подсказку и решение.

  3. Какуро

    Головоломки какуро — это головоломки с перекрестной суммой, похожие на математические кроссворды. Каждое «слово» должно соответствовать числу, указанному в подсказке над ним или слева.

    Скачайте пазлы Какуро 5×5 на KrazyDad. Существуют и другие размеры.

  4. То же число — или вдвое больше

    Вот загадка, которая кажется простой, но является хорошей задачей для критического мышления для младших школьников.Вы можете легко переформулировать это, например, чтобы две стаи птиц сидели на двух деревьях.

    Фермер Браун и Фермер Грин однажды размышляли на заборе между свои фермы. Фермер Браун говорит: «Вы знаете, я просто подумал. Если бы вы дали мне одну из ваших коров, то у нас будет столько же коров ». Фермер Грин отвечает: «Если бы вы дали мне одну из своих коров, я бы имел вдвое больше, чем вы!»

    Сколько коров делает Фермер Браун есть и сколько коров есть у Farmer Green?

    Щелкните здесь, чтобы получить ответ

    У фермера Брауна пять коров, а у фермера Грин семь коров.

    Скрыть ответ

  5. Более двух животных

    Эта головоломка из собрания задач на счеты 2013–2014 гг. Для 3–8 классов церковной школы Грейс. На сайте нет ответов, но студентам предлагается представить свои ответы, и они получат ответ.

    У меня дома более двух животных. Все они собаки, кроме двух. Все они кошки, кроме двух. Все они хомяки, кроме двух.Какие виды животных и сколько у меня каждого животного?

    Посмотреть ответ

    У меня есть три животных: одна собака, одна кошка и один хомяк.

    Скрыть ответ

  6. Вес кирпича

    Это известная проблема, которая звучит так просто, но многих людей обманывает!

    Кирпич весит один килограмм плюс половина кирпича. Какой вес у одного кирпича?

    Вот ответ

    Весит 2 кг. Поскольку кирпич весит 1 кг + половина кирпича, 1 кг дает нам вес другой половины кирпича.Так как половина кирпича весит 1 кг, то кирпич весит 2 кг. Многие неправильно отвечают 1 1/2 кг.

    Скрыть ответ

  7. Среднее количество миль в час
    В гору едет машина. Холм длиной в одну милю. Водитель поднимается в гору со средней скоростью 30 миль в час. Когда водитель достигает вершины холма, он начинает спуск с другой стороны. Сторона спуска также имеет длину в одну милю. Как быстро водитель должен спускаться с холма, чтобы в среднем развивать скорость 60 миль в час?

    Решение

    Невозможно усреднить 60 миль в час.Средняя скорость 60 миль в час требует двух минут, чтобы преодолеть две мили. Первая миля была пройдена со скоростью 30 миль в час. Это означает, что на эту милю ушло две минуты. Следовательно, на вторую милю не остается минут.

    Источник: Math Games and Number Tricks (Ранее опубликовано как Math Puzzles and Oddities) Роберт Явин

    Скрыть ответ

  8. Ноги в автобусе
    1. В автобусе 7 девушек.
    2. У каждой девушки по 7 рюкзаков.
    3. В каждом рюкзаке по 7 больших кошек.
    4. На каждую большую кошку приходится 7 маленьких кошек.

    Сколько ног в автобусе, не считая водителя?

    Решение

    Всего 10 990 ног.
    1. Есть 7 × 7 × 7 = 343 больших кошек. У них 343 × 4 = 1372 ноги.
    2. Есть 7 × 7 × 7 × 7 = 2401 кошечка. У них 2401 × 4 = 9604 ножки.
    3. Есть 7 девушек.У них 14 ног.
    Итого получаем 1372 + 9604 + 14 = 10990 ног.

    Скрыть ответ

  9. Как они могут быть равны?

    Вот небольшая, но загадочная головоломка, которую кто-то прислал на конкурс:

  10. Торговец кокосами
    Умный торговец путешествует с одного места на другое с 3 мешками по 30 кокосов в каждом. Ни один мешок не может вместить больше 30 кокосов. По пути он проходит через 30 контрольно-пропускных пунктов и на каждом контрольно-пропускном пункте он должен дать 1 кокосовый орех за каждый мешок, который он несет. Сколько кокосов осталось в итоге?

    Решение

    Осталось 25 кокосов. Он выдаст по 3 кокоса на каждой из первых 10 контрольных точек и опустошит свой первый мешок. У него осталось два мешка. Затем он раздает по 2 кокаунта на каждой из следующих 15 контрольных точек, опустошая свою вторую сумку. Затем у него остается один мешок кокосов и осталось пройти только 5 контрольных точек, что означает, что он выдает 5 кокосов из последнего мешка и остается с 25 кокосами.

    Скрыть ответ

  11. Емкости для молока
    У молочника есть 8-литровая емкость с молоком, а также две пустые емкости объемом 5 и 3 литра.Ему нужно доставить клиенту 4 литра молока.

    У молочника нет другой запасной тары и нет возможности пометить тару. Он не хочет выливать молоко. Как он отмерит 4 литра молока?

    Решение

    1. Заполните 5-литровую емкость.
    2. Из 5-литрового контейнера вылить в 3-литровый до полного заполнения. В 5-литровом контейнере осталось 2 литра.
    3. Перелейте все молоко из 3-литрового контейнера обратно в 8-литровый контейнер.
    4. Перелейте 2 литра из 5-литрового контейнера в 3-литровый. Таким образом, остается всего один литр свободного места!
    5. Заполните 5-литровую емкость.
    6. Вылейте из 5-литрового контейнера столько молока, чтобы заполнить 3-литровый контейнер (в котором уже есть 2 литра). В 5-литровом контейнере у вас останется 4 литра.

    Скрыть ответ
  12. Фермер и его утки Одного фермера спросили, сколько у него уток.«Ну, — сказал он, — они только что побежали по тропинке, и я увидел одну утку перед двумя утками, утку за двумя утками и утку между двумя утками». Сколько было уток?

    Решение

    Есть три утки, одна напротив другой.

    Скрыть ответ

  13. Числовые головоломки с множеством операций

    Введите каждое из чисел от 1 до 9 в квадраты, каждое только один раз. Порядок действий не действует! Эти головоломки доступны на четырех разных уровнях сложности.

    Пазлы с числами

  14. Пазл для дробей, десятичных знаков и процентов

    Вы можете скачать распечатанную головоломку, чтобы практиковать эквивалентные дроби, десятичные дроби и проценты, опубликованные здесь с разрешения создателя Бекки Колвин. Сначала его нужно вырезать.

  15. Папиросная бумага для складывания
    Лист бумаги толщиной 1/10 мм складываем так, как будто пытаемся уместить его в маленький карман: пополам, еще раз пополам, еще раз пополам и т. Д.

    Предположим, что в уме (потому что на самом деле это невозможно — попробуйте сами!) Мы сложили листок бумаги 50 раз. Насколько толстой будет такая сложенная бумага? Если вы не хотите рассчитывать, по крайней мере сделайте обоснованное предположение!

    Решение.


    Его толщина составляет 1/10 × 2 50 мм.
    Переведя в километры, мы получим, что толщина составляет (1/10) × 2 50 × (1/1000000) км = 112,589,991 км.

    Те, кто не разбирается в этой проблеме, обычно предполагают, что штабель будет не толще пары метров. Однако толщина листка, сложенного пополам в 50 раз, будет на удивление большой, более 112 миллионов километров! (Расстояние от Земли до Солнца около 150 000 000 км).

    Но … сделать это физически невозможно. Длина бумаги, которая вам понадобится для этого складывания, составит световых года … !!! Он задается уравнением

    L = π / 6 · т · (2 ​​ n + 4) (2 n — 1)


    где t — толщина материала, а n количество складок.(Формула Бритни Галливан)

    Скрыть ответ


Еще больше сайтов-головоломок и книг

В приведенном выше списке уже есть много ссылок на веб-сайты с головоломками, но вот еще несколько.

Сложных головоломок отправлено на мой конкурс головоломок.

Пазлы «Москва»: 359 математических увлечений
Эта книга нравится всей нашей семье! Он разделен на 14 глав, каждая из которых посвящена различным видам головоломок, таким как головоломки со спичками, домино и кости, делимость, кросс-суммы и магические квадраты, «Семь раз отмерь, прежде чем разрезать» (головоломки на разрез), «С алгеброй и без нее. «(сложные задачи со словами) и т. д.Головоломки и головоломки варьируются от простых забавных загадок до сложных задач, но ни одна из них не требует продвинутой математики. Они почти всегда иллюстрированы и часто представлены в виде забавных рассказов. Изначально книга была издана в России в 1956 году. Мартин Гарднер отредактировал книгу, чтобы сделать ее как можно более легкой для понимания английской публикой, при этом тщательно сохранив почти всю теплоту и юмор оригинала. Цена: около 5 долларов б / у, около 9 долларов новое.

Математика перед сном
Веселые и оживленные ежедневные математические задачи, сосредоточенные на реальных темах, предлагаются на трех уровнях: маленькие, маленькие дети и большие дети.Вы также можете получить задачи по математике на ночь в бесплатном приложении.

Пазлы Салли с шестиугольными числами
Это «числовые пирамиды», где вы получаете каждое число, складывая числа сразу под ним. Сложность головоломок увеличивается, поэтому в головоломках 4-го уровня используются отрицательные числа, а в задачах 5-го уровня — десятичные дроби.
1 класс — 2 класс — 3 класс
4 класс — 5 класс

Головоломки от ThinkFun
Двенадцать пазлов (PDF) для печати для всей семьи.Большинство из них представляют собой визуальные головоломки, в которых вы переставляете монетки и палочки в другую форму.

MathMania
Детский журнал, полный головоломок, лабиринтов и дразнилок, специально созданных для детей от 7 лет и старше.

Пазлы со спичками
Большой сборник головоломок, в которых нужно переставлять спички.

Пазлы со змеями и шахматами
Вы рисуете змею (или путь) на сетке 5×5 от нижнего левого квадрата до верхнего правого квадрата.В сетке есть несколько шахматных фигур, каждая из которых атакует одинаковое количество сегментов змеи.

MathSphere Математические головоломки
Около двух десятков пазлов для печати (PDF) для младших школьников.

Загадки на математику
Веселые загадки на основе математических понятий.



Математическая логика и задачи со словами, гр.

1-2, электронная книга — Creative Teaching Press ** ЭТО ЗАГРУЖАЕМЫЙ ПРОДУКТ! **

Более 100 воспроизводимых заданий для совершенствования базовых навыков

Предназначен для развития математических знаний и развития навыков критического мышления.Действия сгруппированы по категориям, основанным на стандартах содержания NCTM — Алгебра, Геометрия, Измерение и Анализ данных и вероятность. Также включает:

  • визуальных головоломок
  • задач со словами
  • логических вызовов
  • выкройка
  • многоступенчатые задачи

Серия Power Practice содержит несколько готовых к использованию страниц с упражнениями, на которых студенты могут попрактиковаться. Веселые занятия можно использовать для дополнения и улучшения того, что вы преподаете в классе.Раздайте учащимся страницу с заданием в качестве самостоятельной классной работы или отправьте страницы домой в качестве домашнего задания, чтобы закрепить навыки, полученные в классе. Ключ ответа предоставляется для быстрого ознакомления.

Страницы упражнений в математической логике и задачах 1-2 были специально разработаны для развития математических знаний и навыков критического мышления. Действия включают в себя визуальные головоломки, логические задачи, многоэтапные задачи и многое другое. Все мероприятия способствуют развитию творческого мышления и дают учащимся возможность применять различные стратегии решения проблем.Развитие навыков представлено на нескольких страницах, чтобы студенты могли повторно практиковаться с использованием определенной стратегии.

Действия сгруппированы по следующим математическим стандартам NCTM:

  • число и операции
  • алгебра
  • геометрия
  • измерение
  • и анализ данных и вероятность

Перезарядите умение с помощью Power Practice и дайте студентам возможность добиться успеха!

Рабочие листы-головоломки — Печатные задания-головоломки

Розовый розовый — это пара рифмующихся слов, которые можно использовать для определения глупой подсказки.

4-6 классы

Более простые розовые загадки (рифмующиеся пары).

4–6 классы

Эти розовые тона немного сложнее для детей с более сложным словарным запасом.

с 4-го по 6-й классы

Эти жесткие розовые тряпки действительно потрясут ваш ум и сведут с ума.

4-6 классы

Еще более сложные розовые подсказки, которые нужно разгадывать.

с 4 по 6 классы

Как вы называете большой камень, который холоднее других камней? Более холодный валун! Как насчет пластикового стакана для молодой собаки? Чашка для щенков!

4-6 классы

Как вы называете усталый желтый цветок? Ленивая маргаритка! Как вы называете волшебное существо, которое доит коров? Молочная фея!

4-6 классы

Визуально-пространственные задачи

Какой кусок головоломки подходит для каждой головоломки?

2–4 классы

Назовите цвета, которые вы видите напечатанными на странице, а не слово цвета, которое написано. Эта игра непростая для беглого читателя. (Требуется цветной принтер.)

2–8 классы

Введите числа в калькулятор, переверните его вверх дном и прочтите слова.

Найдите пары чисел в каждой строке, сумма которых равна 10.

Детский сад и 1-й класс

Выясните, какие пары чисел дают в сумме 25.

1-й и 2-й классы

Найдите и обведите пары цифр числа, которые в сумме составляют 100.

1–3 классы

Подсчитайте, сколько вам недель, месяцев и дней.Рабочий лист решения проблем требует навыков умножения более высокого уровня.

3-й и 4-й классы

Квадраты сложения

Математические логические головоломки сложения квадратов

Что я? Вызовы

Используйте подсказки, чтобы определить загадочный объект.

Детектив по номерам

Прочтите подсказки и найдите секретный номер.

Математика с секретным кодом

На этих распечатываемых математических листах учащиеся будут использовать цифровой ключ для декодирования чисел в математических задачах.

Пенелопа Пибоди — 50 штатов

Какой штат посещает Пенелопа Пибоди? Используйте подсказки, чтобы определить ее местонахождение.

Фотографии наших распечаток для головоломок




Головоломки | Ежедневная доза в Интернете 4 Классные комнаты

1. Головоломки и математические головоломки.

Щелкните изображение, чтобы увеличить
Эта страница содержит постоянно растущий список головоломок и математических головоломок по Syvum-Online Education and Interactive Learning. Все головоломки и математические головоломки интерактивны с немедленным подсчетом очков.В головоломках и математических головоломках, а также в их объяснениях используется динамический контент для непрерывного обучения и развлечения. ПОДРОБНЕЕ
2. 17 забавных головоломок для проверки вашего гения.

Щелкните изображение, чтобы увеличить
Несмотря на то, что головоломки являются широко используемым инструментом как в школе, так и в офисе, это не значит, что они не могут быть столь же интересными, как и образовательными. Если вы новичок в разгадывании головоломок или уже просмотрели каждый эпизод Brain Games, вот несколько забавных головоломок, которые испытают ваш гений.ПОДРОБНЕЕ
3. 19 головоломок, которые оставят вас в тупике.

Щелкните изображение, чтобы увеличить
Решение логических задач повышает умственные способности, укрепляет память и развлекает всех до бесконечности. Посмотрите, сколько вы сможете вычислить — без обмана! УЗНАТЬ БОЛЬШЕ
4. 45 забавных головоломок для детей.

Щелкните изображение, чтобы увеличить
Привлекайте учащихся к заданию по языку, математике и визуальным головоломкам.Ответы включены. ПОДРОБНЕЕ
5. Aunty Math.

Щелкните изображение, чтобы увеличить
Каждые две недели для племянницы и племянников тетушки Математика проводятся новые математические задания. Эти проблемы имеют форму сюжетных задач. Ваш класс может опубликовать свои решения и стратегии, а также посмотреть ответы других. Прошлые архивы с ответами. 1–6 классы [Эта ссылка с истекшим сроком действия доступна в Интернет-архиве Wayback Machine.] & nbspПодробнее
6. Brain Bashers.

Щелкните изображение, чтобы увеличить
Сборник головоломок, игр и оптических иллюзий. Этот сайт обычно обновляется как минимум 5 новыми тизерами каждое воскресенье. ПОДРОБНЕЕ
7. Мозговые связки.

Щелкните изображение, чтобы увеличить
Все, что вам нужно, это лист бумаги и мозг. Ссылки, содержащие пазл с готовой картинкой.Ваша цель — сложить пазл в фигуру со сплошным цветом с каждой стороны. ПОДРОБНЕЕ
8. Brain Den — Головоломки и загадки.

Щелкните изображение, чтобы увеличить
Классические головоломки, отсортированные по нескольким разделам; общая логика, взвешивание головоломок, загадки Эйнштейна, числовые последовательности, переход через реку и т. д. ПОДРОБНЕЕ
9. Brain Easer.

Щелкните изображение, чтобы увеличить
Растущая коллекция головоломок, головоломок, загадок и обучения, сгруппированных по сложности / предмету и с подробными объяснениями.ПОДРОБНЕЕ
10. Мозговая еда.

Щелкните изображение, чтобы увеличить
Этот сайт не меняется еженедельно, но есть широкий выбор, который должен занять вас круглый год. ПОДРОБНЕЕ
11. Brain Metrix.

Щелкните изображение, чтобы увеличить
Тренируйте свой мозг; много головоломок и игр. ПОДРОБНЕЕ
12.Брайнгл.

Щелкните изображение, чтобы увеличить
Головоломки, загадки, упражнения, игры, форумы и многое другое … Некоторые из самых крутых функций доступны только зарегистрированным пользователям, но зарегистрироваться можно бесплатно. ПОДРОБНЕЕ
13. Cyberchase.

Щелкните изображение, чтобы увеличить
Игры с критическим мышлением. Они не меняются, но каждое из них представляет собой проблему! ПОДРОБНЕЕ
14.Игры для мозга.

Щелкните изображение, чтобы увеличить
Играйте в бесконечные викторины, развивайте память и развивайте умственные способности, чтобы тренировать свое мышление. ПОДРОБНЕЕ
15. Загадки.

Щелкните изображение, чтобы увеличить
Здесь вы можете найти лучшие загадки для детей и взрослых, легкие и сложные загадки, кто я и забавные загадки и еще много хороших загадок и ответов, которые растянут ваш разум и заставят улыбнуться.ПОДРОБНЕЕ
16. Головоломки Houghton Mifflin, класс 1.

Щелкните изображение, чтобы увеличить
У Хоутона Миффлина есть головоломки для уровня своего класса. Они не меняются ежедневно, но есть несколько тизеров. ПОДРОБНЕЕ
17. Головоломки Houghton Mifflin, класс 2.

Щелкните изображение, чтобы увеличить
У Хоутона Миффлина есть головоломки для уровня своего класса. Они не меняются ежедневно, но есть несколько тизеров. УЗНАТЬ БОЛЬШЕ
18. Головоломки Houghton Mifflin, класс 3.

Щелкните изображение, чтобы увеличить
У Хоутона Миффлина есть головоломки для уровня своего класса. Они не меняются ежедневно, но есть несколько тизеров. УЗНАТЬ БОЛЬШЕ
19. Головоломки Houghton Mifflin, класс 4.

Щелкните изображение, чтобы увеличить
У Хоутона Миффлина есть головоломки для уровня своего класса.Они не меняются ежедневно, но для каждой оценки есть несколько тизеров. ПОДРОБНЕЕ
20. Головоломки Houghton Mifflin, 5 класс.

Щелкните изображение, чтобы увеличить
У Хоутона Миффлина есть головоломки для уровня своего класса. Они не меняются ежедневно, но есть несколько тизеров. УЗНАТЬ БОЛЬШЕ
21. Головоломки Houghton Mifflin, класс 6.

Щелкните изображение, чтобы увеличить
У Хоутона Миффлина есть головоломки для уровня своего класса.Они не меняются ежедневно, но есть несколько тизеров. ПОДРОБНЕЕ
22. Логические загадки.

Щелкните изображение, чтобы увеличить
Посмотрим, сможете ли вы решить эти логические загадки и ответы. Немногие могут получить их все. Проверьте своих друзей, семью, детей и студентов, чтобы узнать, смогут ли они понять это. Дедукция, индукция и умозаключение должны быть в вашем арсенале инструментов, чтобы решать эти сложные и забавные логические загадки с ответами.ПОДРОБНЕЕ
23. Математические викторины.

Щелкните изображение, чтобы увеличить
Есть четыре отдельных математических викторины, каждая из которых содержит новую викторину на каждый день викторины. Это 28 на выбор. Они будут дразнить ваш мозг! ПОДРОБНЕЕ
24. Математические загадки — Math & Logic.

Щелкните изображение, чтобы увеличить
Положитесь на нас в решении математических задач для детей и взрослых! Добавляйте в свой мозг клетки и умножайте веселье с помощью забавных, простых и сложных математических загадок и ответов.Будьте готовы уделить все свое внимание, потому что оно понадобится вам для решения этих забавных математических загадок. ПОДРОБНЕЕ
25. Игры разума.

Щелкните изображение, чтобы увеличить
Играйте в лучшие бесплатные онлайн-игры для разума с играми для разума, математикой, головоломками и словами, судоку и играми на запоминание. В игры можно играть на компьютерах, планшетах и ​​мобильных устройствах (Android, iOS, Windows Mobile). ПОДРОБНЕЕ
26. Решение проблем.

Щелкните изображение, чтобы увеличить
Тизер в день избавит от слабоумия. Тренируйте свой мозг с помощью упражнений на решение проблем (61) из PedagoNet. ПОДРОБНЕЕ
27. Головоломки Prime Brain Teasers.

Щелкните изображение, чтобы увеличить
Здесь вы найдете большую коллекцию некоторых из лучших головоломок. Вы можете фильтровать головоломки либо по сложности — от простого до опытного, либо по теме.Классика, Загадки, Поперечная, Математика, Дедукция, Шахматы, Детектив, Наука, Практика — каждый найдет что-нибудь для себя. ПОДРОБНЕЕ
28. Головоломки Сквигли.

Щелкните изображение, чтобы увеличить
В Squigly есть огромная коллекция головоломок для детей всех возрастов! Эти забавные загадки — способ проверить ваше мышление и навыки решения проблем. Ответы предоставлены, если вы полностью застряли. Сборник разделен на следующие разделы: «Умные головоломки», «Детективные тайны», «Математические игры», «Загадки со стихотворениями», «Пазлы Ребуса» и «Что я задаю вопросы».ПОДРОБНЕЕ
29. Серый лабиринт.

Щелкните изображение, чтобы увеличить
В Сером Лабиринте всегда есть как минимум одна нерешенная загадка. Посмотрите, сможете ли вы сами придумать ответ, прежде чем они его разместят. Перечислены десять недавних головоломок и большой архив головоломок. ПОДРОБНЕЕ
30. The Set Daily Puzzle.

Щелкните изображение, чтобы увеличить
Настоящий вызов для логического мышления. Пазл обновляется ежедневно в 12:00 по тихоокеанскому стандартному времени. ПОДРОБНЕЕ
31. Лучшие головоломки, игры и иллюзии для подростков.

Щелкните изображение, чтобы увеличить
Здесь вы можете насладиться 25 лучшими головоломками, играми и иллюзиями, которые больше всего понравились читателям SharpBrains (в основном взрослым, но и некоторым молодым людям). Всегда полезно узнать больше о своем мозге и потренировать его! »ПОДРОБНЕЕ
32.Коварные загадки.

Щелкните изображение, чтобы увеличить
Эти хитрые загадки с ответами будут ломать голову! Проверьте себя и посмотрите, сколько из этих обманчиво сложных загадок вы сможете разгадать. Помимо коллекции каверзных загадок и ответов, также есть специальный раздел сложных загадок для детей. ПОДРОБНЕЕ
33. Набор для викторин.

Щелкните изображение, чтобы увеличить
Есть девять отдельных викторин, каждая из которых содержит новую викторину для каждого дня недели.Это 63 на выбор. Они будут дразнить ваш мозг! ПОДРОБНЕЕ
34. Игра в слова Викторины.

Щелкните изображение, чтобы увеличить
Есть семь отдельных опросов по словарю, и в каждом из них есть новый тест на каждый день викторины. Это 49 на выбор. Они будут дразнить ваш мозг! ПОДРОБНЕЕ

Математические пазлы для старших классов с ответами pdf

Математические пазлы для старших классов с ответами pdf

математических головоломок для старших классов с ответами pdf Ресурсы для учителя: развивающие игры и головоломки. 6 фев 2019 Головоломки, логические и математические загадки дают учащимся сложные задачи. Align = «aligncenter»] математические головоломки для средней школы pdf 21 мая 2019 г. Включен бесплатный рабочий лист в формате PDF и ключ для ответов. Слова могут быть вперед, назад, по диагонали, по горизонтали. Успех в математике начинается с увлекательных страниц с практическими занятиями по математике. Бесплатно для студентов, родителей и преподавателей. Переход через реку. Com. com www. Математические пазлы для детей — с 1 по 7 классы. 23 августа 2016 г. · Сделайте математику интереснее. Математические головоломки от Льюиса Кэрролла.Избранные ответы на часто задаваемые головоломки: 1000 шкафчиков. Жесткий. 07 августа 2020 г. · Математические загадки и головоломки. Когда мы учились в старшей школе (ребята из математической школы № com, но это не значит, что эти головоломки просты — как раз наоборот — некоторые из них находятся на уровне колледжа. Найдите рабочие листы по координатному графическому изображению. Предположите, что Земля идеальная сфера радиусом 6400 км, и что материал ленты не растягивается.Пазлы для поиска слов можно распечатать, и головоломка меняется каждый раз, когда вы посещаете.Все БЕЗ ПОДГОТОВКИ и удобство печати в формате PDF. 40. Эти головоломки развивают самые разные математические навыки. Используйте математические головоломки в математическом центре. Как доказательство того, что 2 + 2 = 5. Головоломка этой недели Math Cross появляется ниже. Взгляните на 50 простых головоломок по математике для детей, идущих в школу. Ответ: подумайте о средней части торта. Учащиеся могут придумывать собственные головоломки. Участники получают неограниченный доступ к 49 000+ межучебным образовательным ресурсам, включая интерактивные задания, клипарт и генераторы настраиваемых листов abctools.Математические пазлы для детей 1, 2, 3, 4, 5, 6 и 7 классов. → 3. Эти головоломки поддерживают математику, которая важна для учащихся средней школы. Больше математических словесных задач 6 класс · 7 и 8 классы и миддот Что касается математических головоломок, особенно проблем более чем верного англиканца на протяжении всей его жизни, посещающего богослужения Высшей церкви, очень заинтересованного, я добавил несколько сносок к головоломкам и в разделе ответов, если вы даете ученику средней школы чрезвычайно сложные или невозможные математические загадки для решения, это может подорвать его уверенность в себе. Участники получают неограниченный доступ к 49 000+ межучебным образовательным ресурсам, включая интерактивные задания, клипарт и abctools. Программа Smiley Math рассчитана на 10 недель. Упражнения по геометрии также можно превратить в кроссворды по геометрии для другого способа решения математических задач. все разные, которые можно разделить пополам и по продуктам. Отличный набор математических головоломок для старшеклассников. Однако до сих пор движение в этом направлении было ледниковым. Все они легкие. Используйте деление, чтобы ответить на музыкальный Ms.Эти рабочие листы представляют собой упражнения в формате PDF высочайшего качества для печати. распечатайте их и повеселитесь. Используйте свой калькулятор с умом. Сможете ли вы решить эти математические уравнения? задачи, которые читатель должен знать некоторые основные математические концепции на уровне курса математики в средней школе, а также некоторые основные понятия по программированию с использованием языка C. Я просто не хочу, чтобы они были на обоих сайтах. Я часто рассказывал историю из моих школьных лет, иллюстрирующую эту дилемму. 4 июля 2018 г. · Это отличный способ убедить учащихся доказывать и защищать свои ответы, не вызывая угрозы.Эти загадки и головоломки проверит вашу головоломку 5. Включает в себя магические квадраты, числовые шаблоны, поля сложения, судоку и многое другое. 3. MysteryGrid — SolveMe Puzzles 4 декабря 2019 г. · Ответ. Наконец, смоделируйте критическое мышление учащихся к 17 декабря 2015 г. СЛОЖНЫЕ ЗАДАЧИ С МАТЕМАТИЧЕСКИМИ ПРОБЛЕМАМИ, КОТОРЫЕ СТОИТ РЕШИТЬ СКАЧАТЬ НАШИ ЛЮБИМЫЕ ЗАДАЧИ ДЛЯ КАЖДОГО УРОВНЯ Получите наши обучающие онлайн-классы · Рекомендуемые ресурсы для бесплатных загрузок школ Waldorf и семей с домашним обучением. Загрузите и играйте бесплатно в наши распечатанные логические головоломки (PDF).Сколько рукопожатий? Рабочие листы для печати по математике от K5 Learning. Пройдите наши курсы математики в старших классах по преалгебре, алгебре 1, алгебре 2 и геометрии. Поиск слов по геометрии может быть использован для того, чтобы учащиеся освоили новые и незнакомые термины. Математические логические головоломки для детей добавляют дополнительную задачу решения математических задач наряду с логической головоломкой. pdf (7) Процент задач со словами, которые необходимо выполнить: рабочая тетрадь, стр. 106 + 107. Учебный материал ориентирован на математику средней школы США. Карточный магический трюк 3 7.Ответ A или Ответ B C. Толщина 9 сантиметров. 99 Наивысший итог. Абсолютная ценность Bellwork. Каждая ссылка ниже указывает на распечатанный PDF-лист, к которому также прилагается лист ответов. Элдон помогает людям загружать продукты в машины в продуктовом магазине. Посмотрим! Если у вас есть практика тестирования способностей! Только 2% людей решили этот вопрос. Если вам нечего делать в это рождественское время, вот несколько головоломок, которые можно решить. X — нечетное число. Математические головоломки и игры могут быть очень необычными и занимательными.Игры разума. С древних времен у людей всех возрастов есть ключ ответа «Поиск слов по математике». Найдите слова из поля со словами в головоломке и обведите их. подходят для детей в возрасте и старше, а также для старшеклассников и взрослых! Числовая игра математическая головоломка iq головоломка с правильным ответом. Если вы поощряете или позволяете ученикам физически одичать в ваш первый день, это установит ожидание такого поведения на несколько недель вперед. Логические головоломки. Вопросы и ответы. Примерное решение приемлемо.76. Также доступны бесплатные загружаемые Подробные решения (PDF). Каждая из этих таблиц умножения представляет собой SVG с высоким разрешением, поэтому умножение. Как только игра начинается, заключенные не могут общаться, кроме как ответив «красный» или «черный» в ответ охраннику. Цена: 14 фунтов стерлингов. Number Read Online Простые математические головоломки с ответами 1918 39, руководство по разработке программного обеспечения для прессы Роджера, руководство пользователя hx9v, однажды королева, история Элизабет Вудвилл, плантагенет угли, новеллы, книга 2, эксперт f 4 0, jurnal keperawatan soedirman the journal of, grand hotel abyss жизни франкфуртской школы, пользователь binatone veva 1700 Получите бесплатные проверенные детьми рабочие листы для учителей, рабочие тетради, головоломки и ресурсы, которые помогут развить у каждого ребенка знания и любовь к школе. Two Ways x 4 2 5 12 3 8 15 10 120 + Математические игры и головоломки для учащихся средней школы — На веб-сайте Университета Джонса Хопкинса появилась эта страница, на которой представлены математические игры и головоломки для учащихся средней школы. Математика на этой неделе: понедельник: рабочая сессия, студенты ответят на следующие вопросы: (1) WB p. WITRE. Якобы у студентов было всего 20 секунд, чтобы решить задачу! Ответ: 87. Загадки и головоломки Мелочи Менталробика Головоломки Сообщество Логические головоломки Оптические иллюзии Головоломки Коды и шифры Пазлпедия Печатные рабочие листы по науке в средней школе, учебные пособия и словарный запас.Головоломки и задачи также доступны в Национальной стратегии счисления. СБОР МАТЕМАТИЧЕСКИХ ЗАГАДК ГЕНРИ АДАМС. Содержание 1. У нас есть рабочий лист по алгебре, рабочий лист экспонент, соотношение рабочего листа, рабочий лист для решения математических задач и рабочий лист вычитания. Помимо этого, мы предоставили вам головоломки в формате PDF, 50 лучших головоломок в формате PDF и т. Д. Файлы сгруппированы по сложности (очень простые, легкие и средние) и подходят для всех возрастов. Они не могут использовать громкость, интонацию или 20 июл 2017 42 математических тизера для KS3 / 4 — Головоломка на день для летних каникул.Для тех из вас, кто находится в тупике, вот ответы на загадки, а также нешифрованная версия стихотворения Пи Принтера. За одну неделю она заработала 1250 долларов. Все это требует определенной изобретательности, но обычно это только математика до колледжа. Хотя двое мальчиков идут в противоположных направлениях, они идут в школу и из школы по одним и тем же дорогам. Затем добавьте свои параметры (нижняя область) и нажмите кнопку «Создать мой рабочий лист». В первый день он заставляет своих учеников провести необычную церемонию открытия: там тысяча шкафчиков, и один предмет по математике преподается от начальной школы до колледжа.Головоломки и логические загадки играют важную роль, особенно в умственном развитии старшеклассников. Логическая головоломка — это задача, задача или игра, требующая от игрока использования форм тестов по математике, созданных в зависимости от уровня обучения и согласованных с Общей основной учебной программой по математике. Уберите алфавит от X, и он станет четным. Эти распечатанные рабочие листы по математике для средней школы отлично подходят для любого класса. На каждом листе уже нарисована сетка данных, чтобы дети могли практиковать свои навыки дедуктивного мышления.Посоветуйте учащимся сформулировать способы, которыми они могут скорректировать свои стратегии критического мышления с учетом следующих задач, которые они решают. com: Учебный центр математики разрешает классным учителям воспроизводить черных линий мастеров в соответствующих количествах для использования в классе. Подростки в старшей школе склонны легко терять концентрацию из-за разнообразия обстоятельств, которые происходят каждый день. Пазлы для печати логической сетки. Теперь добавьте 40. 5 мая 2010 г. · Математические пазлы Пазлы опубликованы в блоге математики старшеклассников. Комментарии: Ответ 28 4.Геометрия — Фигуры: создайте квадрат из квадратов в пазле — для дошкольного и детского сада. Предоставлено ThinkFun. Математика. Математический проект Нрича, Кембридж, Англия. 4 сантиметра в высоту, 72. Найди сложную игру для мозга: найди сложную математическую головоломку с наибольшим числом и ответом. Ten Hats 3 10. Получите помощь онлайн или офлайн. Ответы. МАТЕМАТИЧЕСКАЯ СРЕДНЯЯ ШКОЛА С КНИГой PIZZAZZ1 C. Математические логические головоломки для детей. Все дело в логическом мышлении, поиске закономерностей и связей и решении проблем. математические настольные игры.Любители математики найдут то, что им нужно для удовлетворения своих пристрастий, в следующих математических загадках от GPuzzles. Щелкните здесь, чтобы увидеть головоломки Math Cross из предыдущих недель. Хороший способ привлечь внимание класса — использовать сложные загадки, связанные с предметом, который вы преподаете, или использовать головоломки, которые соотносят рабочие листы бесплатных математических головоломок в формате pdf для печати, рабочие листы математических головоломок для практики и улучшения различных математических навыков, сложения, вычитания , соотношения, дроби, деление Вы столкнулись с проблемой на экзамене с выбором только одного ответа, вопрос был пропущен. Эти ресурсы могут служить дополнительными материалами TEFL в школе, а также дополнительными домашними заданиями для детей, обучающихся на дому, по математической лексике и определению слов. Предлагается широкий ассортимент тем, включая логические игры, числовые головоломки и даже математические шутки! Изучите математику в пятом классе в соответствии с учебной программой Eureka Math / EngageNY — арифметику с дробями и десятичными знаками, задачи с объемом, преобразование единиц измерения, точки на графике и многое другое. Некоторые головоломки оригинальны. Задача недели (POW) — это увлекательная математическая задача, доступная для всех учащихся.Вы можете проверить свой уровень знаний, решая сложные математические головоломки и ответы на них. ICESTAAO. Все рабочие листы имеют ключ ответа на 2-й странице файла. В некоторых случаях вам также удастся не обнаружить сообщения Тысячи математических головоломок, чтобы заставить ваш разум думать. 6 октября 2016 г. · Пирамиды — забавная и обманчиво сложная головоломка. Ih opeyou! Наслаждайтесь! Используя! Эти! Math! Brain! Тизеры. Читать онлайн Простые математические головоломки с ответами 1918 39, руководство по разработке программного обеспечения для прессы Роджера, руководство пользователя hx9v, однажды королева, история Элизабет Вудвилл, плантагенет угли, новеллы, книга 2, эксперт f 4 0, jurnal keperawatan soedirman the journal of, grand hotel бездна жизни франкфуртской школы, пользователь binatone veva 1700 Квадраты, на которых начинаются ответы, обычно нумеруются.У нас есть более 2500 забавных и сложных математических головоломок с решениями. 19 -17 — 7 Раскрасьте этот ответ в желтый цвет. Всего используется более 2500 различных слов, большинство из которых часто встречаются 26 августа 2020 г. · Проверьте свою логику с помощью 25 логических головоломок, включая простые головоломки на словесную логику для детей и сложные логические головоломки для взрослых. С 1802 года Розеттский камень хранится в Британском музее в Лондоне, Англия. 15 мая 2017 года студенты второго курса инженерной математики. 26 ноября 2018 г. · 1-7 Свойство распределения 7-1 Нулевые и отрицательные экспоненты 8-2 Умножение и разложение на множители 10-2 Упрощение радикалов 11-3 Деление многочленов 12-7 Теоретические и экспериментальные вероятностные уравнения и неравенства абсолютных значений Алгебра 1 Игры Алгебра 1 Рабочие листы обзор алгебры решение уравнений лабиринт ответы Синко де Майо Математическое задание Эти простые математические головоломки с ответами оказываются большой проблемой для тех, чей ум не привык к работе.Каждую пятницу учащиеся загружают рабочий лист заданной недели с веб-сайта нашей школы в pembrokelakes. Рабочий лист. Вот отличная коллекция полезных головоломок для школьного класса. Внимательно прочтите каждый вопрос, чтобы убедиться, что вы понимаете тип требуемого ответа. com). Головоломки всегда интригуют школьников и, следовательно, являются отличным средством мотивации к изучению математики. Они охватывают геометрию, вероятность, теорию чисел, алгебру, исчисление, тригонометрию и логику. L. Каждая подсказка состоит из двух слов, которые рифмуются; все, что вам нужно сделать, это ввести правильные буквы. Для просмотра и печати файлов PDF вам понадобится бесплатная программа Acrobat Reader. Например, KCART будет обратным отсчетом, поскольку это слово «дорожка» написано в обратном порядке. com. MCATA, NCTM — задачи с низким полом, высоким потолком и множеством точек входа, позволяющие всем студентам. Департамент математики 2018: индивидуальный экзамен (PDF), решения для индивидуального экзамена (PDF), 2001: индивидуальный экзамен, проблемы с шифрованием (PDF). Вот рифмующаяся головоломка, чтобы проверить свой мозг.Mashup Math LLC 2020 | Больше бесплатных математических загадок, головоломок и заданий можно найти на сайте www. Если вы решите использовать калькулятор, убедитесь, что он разрешен, работает в день проверки и имеет надежные батареи. 3 сантиметра в ширину и 27. Они являются отличным способом обзора материалов по естествознанию и математике. Математический кроссворд №33 Разделение на простые остатки. Математический кроссворд № 34 Различные математические формулы и измерения. Слишком много солнца заставило ее уши хрустеть. Учителя и домашние школьники используют рабочие листы по математике на этом веб-сайте для измерения рабочих листов по математике для учителей в начальной, средней школе и детском саду. Используйте бесплатные рабочие листы по математике в формате PDF для выполнения домашних заданий и закрепления концепций, навыков и пакетов — вашим ученикам нужен быстрый ресурс для практики их математические навыки. Математика — учеба и преподавание (среднее образование) — Манитоба.Загружаемые файлы PDF для всех классов. Обзор учебной программы по математике — 1–12 классы. Proj Geom — Головоломки с решениями. Домашнее задание по математике = без объяснений и восемь задач за ночь. Это была бы прекрасная альтернатива дидактическим карточкам для усвоения математических фактов. Щелкните здесь, чтобы загрузить версию в формате PDF и решение. 26 ноября 2018 г. · 1-7 Свойство распределения 7-1 Нулевые и отрицательные показатели 8-2 Умножение и разложение на множители 10-2 Упрощение радикалов 11-3 Деление многочленов 12-7 Теоретические и экспериментальные вероятностные уравнения и неравенства с абсолютными значениями Алгебра 1 Игры Алгебра 1 Рабочие листы обзор алгебры решение уравнений лабиринт ответы Синко де Майо Математическая деятельность Ваш мозг всегда учится, и школа — это ваша лучшая возможность сосредоточиться на расширении своих знаний и умственных способностей. 4. Действительно понятные уроки математики (предварительная алгебра, алгебра, предварительное вычисление), классные математические игры, графические онлайн-калькуляторы, геометрическое искусство, фракталы, многогранники, области для родителей и учителей. Письмо укрепляет изученную математику. Из второго разреза, пересекающего первый, получится четыре части, а из третьего разреза можно заглянуть в наши Кроссворды Numbers и сыграть или распечатать свою головоломку сегодня! Если вы любите математику, то у нас есть лучшие онлайн-кроссворды для вы, и они свободны. Манитоба. Средняя серия по математике · Предалгебра · Алгебра. У них есть один набор процедур для решения задач вне школы и в старшие классы, когда учащиеся в первую очередь выполняют выбор в рамках учебных целей.Укладка шахматной доски 3 4. Хотите верьте, хотите нет, но этот «математический» вопрос на самом деле вообще не требует математики. Еще одна простая в использовании математическая головоломка, которую дети будут любить делать, если 23 = 44 = 58 13 = 23 = 45 45 = 11 = 92. (Источник: Noetic Learning) Мы сотрудничаем с Noetic Learning, чтобы предложить вам «Проблема Неделя »! Используйте эти интересные и нестандартные творческие математические задачи, чтобы помочь вашим ученикам мыслить логически, творчески и математически. Домашняя страница Хорхе Нуно Сильвы Головоломка с математическим крестом №24 Деление (на одно- и двухзначные числа, без остатков) Головоломка с математическим крестом №25 Уравнения, измерения и деньги.Детям это тоже нравится. Эти головоломки предназначены для проверки на числовые способности, логическое мышление, решение математических задач с помощью вопросов и ответов. Угадайте 21 июля 2020 г. Математические загадки для детей. Используйте приведенные ниже бесплатные головоломки по алгебре, чтобы помочь своим более вербальным ученикам понять алгебраические концепции. Математические загадки Эти загадки так же забавны, как и приведенные выше, но они ориентированы на математику. 20 3 — 24 Раскрасьте этот ответ в фиолетовый цвет. Студенты будут использовать все основные навыки оператора, о которых вы только можете подумать. Решите эти проблемы со словами, включив ответы.Работая с одноклассником, учащиеся могут поделиться своими знаниями английского языка при решении различных лексических или грамматических вопросов, поставленных в головоломке. 42! Это ответ на вопрос «сколько дней в шести неделях?» а также «Жизнь, игра престолов с одним глазом и планирование» Найдите недостающие цифры, а затем поместите свои ответы в соответствующие поля. Математические загадки Следующие математические загадки наверняка доставят вам трудные времена. Многие классические математические доказательства парадоксов (иногда называемые «пуфами») зависят от такого рода обмана, такого как скрытое деление на ноль.I. В этом бесплатном одностраничном листе учащиеся решают различные головоломки. Такие вопросы, основанные на головоломках, часто задают на различных конкурсных экзаменах и собеседованиях. Добро пожаловать на страницу математических заданий Word Problems со страницы Рождественских заданий по математике на сайте Math-Drills. Однако, поскольку математика одинакова во всем мире, мы приветствуем всех. Тонны математических и логических задач, которые вы можете решить. Эта ссылка ведет вас на Coolmath5kids. Это средний раздел — 10 головоломок. → 2.В сложных головоломках уже есть меньше чисел и требуется больше стратегий. Сама логика — это стиль мышления, который должен использоваться во всех областях математики. Для тех из вас, кто является математическим гением, вот прощальная загадка, придуманная не кем иным, как самим великим Альбертом Эйнштейном! Подобные математические головоломки не связаны с буквальным толкованием математических символов. Сто кварталов 3 5. Они отлично подходят для разминки в классе или после экзамена. Когда они встречаются, кто был ближе к их дому? Ответ на головоломку Этот набор логических головоломок на День Благодарения включает в себя 1 головоломку с сеткой, 2 головоломки судоку и ключи для ответов.Снова в школу — поиск слов. 12 + 6 + 2 = 20. 26 февраля 2009 г. · См. Загадку 160. Введение. 17 июня 2020 г. · Обширная база данных из более чем 19 онлайн-тестов по математике для старших классов. Проверьте свои знания с помощью вопросов викторин по математике для старших классов. Они либо определяют недостающие значения, либо находят число, не входящее в группу. Раскрасьте этот ответ в желтый цвет. Разгадывайте все типы головоломок и будьте уверены в себе к экзаменам. подчеркнула за вас первую букву каждого слова). Доджсон (Льюис Кэрролл — псевдоним английского писателя и математика Чарльза Лютвиджа Доджсона, особенно известного по детским книгам «Приключения Алисы в стране чудес» и «Зазеркалье»).Все рабочие листы представляют собой документы в формате pdf с ответами на 2-й странице. подробнее >> Учебные материалы для K-12 (Архив математики) — Университет Теннесси, Ноксвилл (UTK) Автобусные билеты пронумерованы шестизначными числами, а в счастливом билете сумма первых трех цифр равна сумме трех последних. цифры. Если его комиссия составляла 8 880 долларов, какова была продажная цена дома? По математике помогает с 1998 года. Головоломки, задачи и вопросы в этих уроках интересны и интересны сами по себе. Найдите здесь сотни бесплатных кроссвордов по геометрии или составьте свой собственный кроссворд по геометрии! Кроссворды по геометрии помогают учащимся понять и запомнить важные геометрические термины.50 в час плюс чаевые. Даны все ответы. Щелкните, чтобы открыть версию для печати в формате PDF (формат документа для печати). Методики обучения. Три петли 3 8. k. Ученики Грэма (@GraemeAnshaw) изучают площадь, дроби и проценты с помощью математических рождественских елок и ламп для меноры. Ответ А. Б. Рабочие листы на этом сайте созданы в формате PDF. Математические головоломки могут быть очень необычными и занимательными. Эти простые математические головоломки с ответами оказываются большим испытанием для тех, чей ум не привык к работе.Посетите наш раздел классных математических игр, чтобы получить отличный способ пересмотреть свою математику. Многие учителя используют их как еженедельные задания. Привлекайте своих учеников с помощью этих распечатываемых рабочих листов по решению задач для младших классов средней школы. НАОЗИГРЭ. com — это онлайн-ресурс, которым ежедневно пользуются тысячи учителей, учеников и родителей. В этом разделе вы можете разучивать и практиковать логические, числовые, словесные, математические и т. Д., Поэтому можно составить четыре различных числа. Щелкните изображение, чтобы увеличить: играйте в лучшие бесплатные онлайн-игры для разума с играми для разума, математики, головоломками и словами, судоку и этой книгой, которую можно использовать в качестве учебного пособия в 7, 8 или 9 классах (обычно в течение двухлетнего периода) или как справочник для старшеклассников.Вот ответы: A. 6. Цель головоломки — определить правильные комбинации. Это листы-раскраски по математике на День Благодарения, поэтому вам понадобится что-нибудь, чтобы они могли раскрасить свои листы с ответами. дроби могут помочь учащимся решать задачи в виде словесных задач или повседневных задач. Наибольшую трудность для учащихся составляли операции между уровнями многопользовательской школы. Некоторые студенты Токио посещают дополнительные вечерние или субботние занятия, чтобы подготовиться к вступительным экзаменам в лучшую частную среднюю школу. , если конкретный ответ отсутствует в коде поиска ответов или выполнения другой механики головоломки.Содержание Есть 9 головоломок, от простых до средних до сложных. эластичный. Похожая головоломка 28 Глава 11. Он весит примерно 1676 фунтов. Моя первая цифра на 3 меньше, чем моя. Каждая головоломка основана на определенной теме, и большинство, если не все, ответы связаны с этой темой. Если у вас его нет, вы можете получить его здесь. Die Magic Trick 3 6. Партнерство школы и университета для обновления образования по математике. Финансируемые NSF аспиранты STEM в образовательном проекте K – 12 Гавайский университет в Маноа, факультет математики. Логические головоломки: благодарность за выполнение функций. Благодарность Берту Каннеру, Специалисту SK Online по математике и Джиму Саффилсу, SK Online за поддержку веб-страницы. В результате ваша скорость решения головоломок повысится автоматически.Эти рабочие листы мы рекомендуем раздавать ученикам младших классов средней школы. Мы упорно работаем, чтобы убедиться, что наш веб-сайт Квадраты, в которых ответы начинаются обычно пронумерованы. Эти головоломки представляют собой веселые занятия для детей и охватывают несколько математических тем в ранее упомянутых классах. Использование математических головоломок в классе. Баннекер не использовал символическую алгебру для решения этих задач, и они. Тем не менее, люди, которые закончили математику в колледже и старшей школе, также получат ответы на все головоломки и на те игры, которые требуют ответов: В старшей школе есть странный директор. .Окружные программы, мероприятия и практики в любом окружном офисе, школе или школьном мероприятии должны быть свободны от дискриминации, включая дискриминационное преследование, запугивание и запугивание, направленные против любого ученика или сотрудника со стороны кого-либо, на основе реальных или предполагаемых математических головоломок. Пазлы Последовательности / Серии Наборы Квадратные / Кубические корни Статистика Трансцендентные числа Тригонометрия: Обзор головоломок для старших классов Звезды указывают на особенно интересные ответы или хорошие места для начала просмотра. Моя вторая цифра в 4 раза больше третьей.Рабочий лист должен быть возвращен их учителю в следующую пятницу, за некоторыми исключениями (см. Расписание ниже, чтобы узнать точную информацию о геометрических формах: формирование круга (сферы), головоломки для дошкольного и детского сада. Все наши задания являются оригинальными, Проверено в классе, соответствует возрасту и создано опытными учителями, сертифицированными Национальным советом. 19 декабря 2002 г. • Насколько высоко вы тогда были бы над поверхностью Земли? То есть найдите h на диаграмме ниже. Наши викторины по математике для средней школы онлайн может быть адаптирован в соответствии с вашими требованиями для прохождения некоторых из лучших школьных математических викторин.Обзор, обучение, развитие логических и пространственных навыков. Рабочие листы генерируются случайным образом, поэтому каждый раз вы получаете разные. Математика 27 сентября 2018 г. · Вы можете выбрать для своего ребенка его в соответствии с его способностями. Напишите свой ответ на линии. 05 марта 2012 г. · 114 слайдов, поощряющих использование математики для решения задач. RENNECATCTO. ! ! Подробнее! Math! Brain! Teasers! Доступны! В !! Математические задачи! И математика! Головоломки! Набор А! 54 математических головоломки Карточки в 2 различных форматах — большой размер 2 на листе и — маленький размер 6 на листе, идеальный в качестве рабочего листа. Интересный способ развить математические навыки, навыки мышления и проблемные навыки.Ожидание бесплатного PDF-файла с вопросами для рассуждений с новыми шаблонами завершилось. Я трехзначное число. Иногда это невозможно сделать без риска — поощряйте угадывать и проверять, а также извлекать уроки из неизбежных ошибок, особенно при решении более сложных головоломок. Рабочие листы с логическими головоломками и рабочие листы с загадками обязательно улучшат навыки критического мышления вашего ребенка. Марси Кук Математика — Математические карточки и книги для старшеклассников Снова в школу — Поиск слов. Пазлы с буквами и цифрами. Математика может быть интереснее, чем ожидалось! ДОБРО ПОЖАЛОВАТЬ, IGCSE | Доктор Клеменс в прошлом преподавал математику в средней школе. Его можно распечатать, загрузить или сохранить и использовать в вашем классе, домашней школе или другой образовательной среде, чтобы помочь кому-то. Билеты на автобус пронумерованы шестизначными числами, а в счастливом билете сумма первых трех цифр равна сумме. из 3 последних цифр. Математический крест № 30 MathSphere Математические головоломки. T PETREA IUZAVISLE Головоломки для старшей школы — отличный учебный ресурс для тех, кто занимается образованием.Ключевая концепция: шаблоны и головоломки — отличная альтернатива тому, чтобы помочь учащимся обрести уверенность в решении задач. Математика — это вовсе не запоминание формул или выполнение процедур снова и снова. Раскраски с английским алфавитом Проследите свое имя Рабочий лист Детский сад Рабочие листы на аудирование Рабочие листы средней школы Рабочий лист по химии детский рабочий лист математические уравнения Математика для 1-го года Бесплатные рабочие листы Грамматические упражнения Рабочие листы Средняя школа решает проблему со словами на листе моих детей Учебник английского решает уравнения с целыми числами Рабочий лист Хелен Келлер Бесплатная бесплатная математика 16 ноября 2017 г. · Алгебра в формате PDF с головокружительной целью 3 e ключ к ответам 28 страниц из «Алгебра с ответами на лист Pizzazz», источник: node60381-propdf.с другими учащимися … в отличие от абсолютного творчества, которое оценивается с помощью сложных головоломок, необходимые знания простой школьной математики, например, математические загадки для детей, помогут вашим детям тренировать свой мозг и улучшать творческие способности и логическое мышление. Чтение книг на математическую тематику может стать отличным способом побудить детей более активно заниматься математическими задачами. Убедитесь, что вы ответили на заданный вопрос. бровишколы. Сто лампочек 4 14 направлений Найдите ответы на эти проблемы.Обязательно изучите разные вкладки для разных типов головоломок и загадок. Фрэнк уезжает из школы на спортивном автомобиле, а Джек на велосипеде покидает их дом. «Мосты по математике» — это стандартная учебная программа для K – 5, которая представляет собой уникальное сочетание словесных задач по математике для 10-го класса с ответами и решениями. Предварительный просмотр Распечатать ответы 26 февраля 2009 г. · См. Задачу 160. ПОДРОБНЕЕ: 25. 17 февраля 2012 г. · Задачи различаются по сложности, поэтому я позволяю ученикам задавать вопросы на каждом листе в любом порядке — первая команда получает все правильные победы! Ответы включены, так что вам просто нужно распечатать и идти! Помните, что если вам нравятся мои ресурсы TES, вы должны получать мой еженедельный информационный бюллетень по математике (для подписки просто напишите по адресу aap03102 @ gmail.6 декабря 2018 г. · Детям постарше может понравиться ее серия рождественских математических и логических головоломок. D. Вы также можете сформировать их перевернутые формы: 8167294305 и 4927618305. com Times Tiles 7 3 8 6 3 8 Как и математические головоломки, эти головоломки для детей могут улучшить взаимодействие с математическим содержанием и вдохновить ваших учеников на работу над математическими концепциями и задачами вне рамок. регулярные уроки. net Worksheet Pre Algebra with Pizzazz Answer Key Creative from Algebra With Pizzazz Worksheet Answers, источник: cathhsli. # 3 — Загадка сложной алгебры. Возьмите число 1000 и прибавьте к нему 20.Юмор положительно влияет на моральный дух класса. Включение обучающих игр и головоломок в сегодняшнюю школьную программу — отличный способ заложить основы языковых и математических навыков. 63, что очень много по сравнению с другими. Я использую здесь термин «A-level» как сокращение для типичных загадок, головоломок и логических задач с ответами. Пополняйте свой словарный запас английского языка с помощью предлагаемых нами мероприятий. На каждом плакате изображена сложная загадка или логическая головоломка, над которой учащиеся и сотрудники будут размышлять весь день.Веб-сайт Тайеба Загрузите этот набор из 20 плакатов для вашего класса или холлов средней школы. Проверьте свои логические и расчетные навыки с помощью этих числовых головоломок. Попробуйте использовать математические головоломки, чтобы научить учащихся думать о математике. Решите эту лучшую логико-математическую головоломку. Такие головоломки иногда называют дурацкими словечками или ребусами, хотя последние обычно представляют собой головоломки, основанные на картинках. Шармейн работает 30 часов в эксклюзивном ресторане. Предоставляются подробные решения и ответы на вопросы.Таблицы доступны как в формате PDF, так и в формате html. Помимо юмора, интерес учащихся привлекают разнообразие и новизна процедур решения головоломок. Головоломки и задачи также доступны на листах для печати в формате PDF с функциями национальной стратегии счисления, головоломками для поиска слов, соответствием орфографии, зашифрованными предложениями, перекрестными словами и многим другим. Если вы не можете их выполнить, спросите своего директора! (Они могут настолько увлечься, что забудут о задержании, которое вы получили!). Сложные математические головоломки №1 — Нищие не могут выбирать Сложность Популярность Просмотр Ответ Категория обсуждения: МАТЕМАТИКА №2 — Головоломка с семью цифрами 21 октября 2014 г. · A + B = см.Результатом стал сборник — список головоломок, подобранный учителями и родителями! На этой странице перечислены наиболее сложные головоломки. Smiley Math можно найти на вкладке School Forms. НОВИНКА! В своем первом в году задании по математике задайте хороший тон. Режем пирог. Тренируйте математическую часть своего мозга. Эта книга охватывает не только Национальные стандарты математики в средней школе. Некоторые из них очень сложные! Ответы выделены (очень) мелким шрифтом внизу каждого плаката. Многие учителя используют их. Если вам нравится то, что вы видите здесь, вам понравятся все наши рабочие листы по математике 1-го года в цифровом формате.Математический кроссворд № 31 Измерение времени (годы, десятилетия, века) Математический кроссворд № 32 Различные математические формулы и измерения. Например, есть 4 девушки. ) Загадки. Наши головоломки и головоломки подходят для нескольких школьных предметов, будь то словесная лестница для помощи в написании, игра на сопоставление форм для математики и геометрии или кроссворд с уравнениями, чтобы смешать и то и другое! Хотя они могут показаться развлечением, на самом деле они созданы нашей командой экспертов в области образования. Математические загадки и головоломки — отличный способ провести свободное время студентов.Отличный набор математических головоломок для детей старших классов. 00 в подсказках. Математический крест № 29 Расчет средних значений. Р. Или если только они хотят узнать об этом больше. В этом кроссворде также есть необязательный флажок для ключа ответа, который вы можете выбрать, если хотите быстро найти ответы или распечатать их. С помощью этого блога я ставлю перед собой задачу научить других учителей создавать привлекательные и увлекательные классы, в которых изучение математики доставляет удовольствие. Три шляпы 2 3. Генератор математических листов — создавайте свои собственные арифметические, алгебраические, сравнительные, порядковые и округляющие рабочие листы.Эти головоломки для поиска слов «Снова в школу» содержат скрытые слова «Снова в школу», которые нужно найти, и картинку, которую нужно раскрасить. Дьявольский и демонический начальник. СЕЙЧАС самое время сделать сегодняшний день первым днем ​​вашей оставшейся жизни. Все эти вопросы часто используются на всех экзаменах. так что работайте больше над головоломкой высокого уровня и pdf расположения сидений. Делайте головоломки проще или сложнее, используя множество вариантов дизайна. Получите это бесплатно от Math Geek Mama здесь! Мы ценим вашу конфиденциальность и обещаем никогда не отправлять вам спам; Вы можете отписаться в любое время.Математический кросс № 26 Уравнения, измерение и деньги. Математический кроссворд № 35 Различные математические формулы и измерения. Для школьников, учителей и родителей. Нечестная монета 4 12. Раскройте свои большие идеи Математика: Алгебра 1 PDF (Глубокое динамическое исполнение) сегодня. Партнерство школы и университета для обновления образования в области математики. Финансируемый NSF научный сотрудник STEM в образовательном проекте K – 12 Гавайский университет в Маноа, факультет математики. Логические головоломки: благодарность за функции. Благодарность Берту Каннеру, онлайн-специалисту по математике SK, и Джиму Saffeels, SK Online за поддержку головоломок на веб-страницах. Она зарабатывает 3 доллара. Эти распечатанные рабочие листы для решения задач средней школы для младших классов отлично подходят для любого класса. Найдите свое решение среди вариантов ответа. Что садовник ставит в саду в первую очередь? Его или ее ноги. Наши бесплатные рабочие листы по математике охватывают весь спектр математических навыков в начальной школе, начиная с чисел и заканчивая счетом до дробей, десятичных знаков, задачами со словами и многим другим. В учебном году мы не сможем создавать новые задачи недели для математиков. Используйте как «забавное» домашнее задание. В этой книге рассматриваются вопросы организации, планирования и обучения класса посредством ответов на часто задаваемые вопросы.Знание соответствующих формул является обязательным условием для учащихся от 6 класса до средней школы для решения некоторых из этих рабочих листов в формате pdf. Задачи, решения для детей Предложите своим ученикам решить задачи на сложение с помощью 4- и 5-значных чисел. На этой странице указателя вы найдете ссылки на десятки тем по математике в средней школе на этом сайте. Продукты 1 — 255 Тысячи распечатываемых листов по математике для всех классов мгновенно распечатываемые и готовые к использованию в классе или домашней школе. Они идеально подходят для цветной печати и ламинирования, что делает их долговечным ресурсом.Sink the Sub 3 9. математик, C. Привлекайте своих учеников с помощью этих распечатываемых рабочих листов по математике для средней школы. Сделав зашифрованные списки ответов короткими, а процедуры простыми, мы постарались свести к минимуму время, затрачиваемое на поиск ответов или выполнение других задач. Начиная от сложения и заканчивая переменными, игровые головоломки обязательно станут отличным занятием в классе. одного и того же уровня, и всегда полезно обратиться за помощью к кому-то на более высоком уровне. У них намного меньше учеников, чем учителей в других школах, они никогда не меньше ориентированы на запросы, привязаны к учебной программе, дифференцированные математические задачи для 8-го класса Джесси Ширли и Отделение католической школы Гранд-Прери. 17 декабря 2019 г. · Математические загадки и числовые головоломки, которые бросают вызов вашему нестандартному мышлению. Саксонский курс математики 5 Ответы Страница 3 Практические числа по письму 1–100 Задания по умственной математике для 6-го класса Задачи по математике для печати 2-й класс Перегруппировка заданий Трансформация функций Рабочий лист Ответы Алгебра 2 Тест по математике в первом классе математика th рабочие листы по математике ks2 год 5 для печати простое дополнение интерактивные игры математическая головоломка пазл экономика репетитор этап 2 рабочие листы по математике тест по бизнес-математике Печатные задания по математике от K5 Learning.Preview Print Answers Избавьтесь от социальных и культурных нарративов, сдерживающих вас, и позвольте поэтапным решениям из учебника Big Ideas Math: Algebra 1 переориентировать ваши старые парадигмы. Ежедневное математическое задание — Школа Филлипса Брукса — это независимая дневная школа с совместным обучением, предлагающая обучение, действительно ориентированное на ребенка. Первая страница PDF-файла: Образец видения Вселенной Высота горы составляет около 3600 футов. Взгляните на нашу логику и загадки для печати. В настоящее время я преподаю алгебру 2, предварительное исчисление и статистику в средней школе Coweta в городе Коуэта, штат Оклахома.Поиск слов — отличное занятие, которое поможет вашим детям развить навыки решения проблем и аналитические навыки. Головоломки, требующие особой осторожности при ограничении до нуля и бесконечности. 0 — наименее распространенная цифра, хотя в 1000 три нуля! Разъяснения к обеим загадкам 16 декабря 2015 г. — это забавный раздаточный материал, полный головоломок для учащихся 4–6 классов. Решение головоломки судоку, как и логических головоломок, поможет учащимся развить математический образ мышления, даже если они не решают математические задачи.Головоломка 5. Адвент Угадай Рождественские гимны в мире Викторина Сможешь ли ты определить эти рождественские песни Викторина Праздничный кроссворд Сколько ты знаешь викторина Рождественские книги и авторы Рождественские слова в словарной игре Угадай заглавия в песнях Викторина Рождество… Используйте эти математические головоломки для поиска слов, чтобы познакомить школьников со словарем и терминами с новыми математическими концепциями! Эти головоломки для поиска слов включают наборы для различных уровней обучения Common Core, а также конкретные темы по геометрии, алгебре и многому другому! Прокрутите вниз, чтобы найти ссылки для загрузки 7 различных PDF-файлов с упражнениями по раскрашиванию геометрии на День благодарения. makemathmorefun. Ваша задача состоит в том, чтобы анализировать картинки, чтобы преобразовать части в новые формы в следующих головоломках для мозга. Эти рабочие листы по математике для детей содержат упражнения по предварительной алгебре и алгебре, подходящие для дошкольных учреждений, детских садов, от первого до восьми классов, бесплатные рабочие листы в формате PDF, рабочие листы по математике для 6-го класса. Ответы на большинство задач прилагаются. Найдите вопросы и ответы с числами. Затем найдите номер ОТВЕТА на раскраске и раскрасьте его цветом, указанным в поле.PDF | При преподавании математики в Чешской Республике она традиционно преподавала серию об экспоненциальных функциях группе старшеклассников. Эта книга содержит ответы. Представленные здесь математические головоломки выбраны из-за обманчивой простоты их постановки или изящности решения. 6 октября 2016 г. Если Google Home может ответить на этот вопрос, то это не критическое мышление высокого уровня. Мы также подготовили практические тесты для SAT и ACT. 6 декабря 2017 г. · Загрузка также включает в себя ключ с ответами, что делает его веселым и недорогим подготовительным рождественским математическим заданием для детей средних и старших классов.Модуль 1: Числовое значение и десятичные дроби: 5-й класс (Eureka Math / EngageNY) Этот набор логических головоломок на тему Дня благодарения включает 1 головоломку с сеткой, 2 головоломки судоку и ключи для ответов. 1. Головоломки с поздравительными открытками на умножение: сделайте забавную головоломку с листом приветствия на умножение, которую нужно решить. Полезно для запоминания фактов умножения. Эта страница преобразует ваши приветствия / сообщения в арифметические задачи. COSFU. Сложная проверка орфографии, математики и чтения распечатываемых рабочих листов в формате PDF, которые вы ХОТИТЕ распечатать для своего класса и детей! В феврале 2014 года я попросил своих подписчиков назвать их любимую математическую головоломку.Головоломки и загадки с ответами для собеседований и вступительных испытаний. правильные ответы организации, а не учителя • Учащиеся используют разумные методы решения Математика Квадраты 12 15 8 50 9 1 1 6 4. Теперь добавьте 1000 еще раз. 7 может вспомнить это) нам нужно было написать код, который распечатывает номера всех счастливых билетов; по крайней мере, я сделал это, чтобы показать свою лояльность клану программистов. Подсказка — ответ — решение Попробуйте решить головоломки самостоятельно, прежде чем смотреть на решение в конце страницы. Головоломки на этой странице являются разминкой.Ниже приведены распечатанные рабочие листы с математическими играми и головоломками для старшеклассников. Одним прямым разрезом вы можете разрезать пирог на две части. Кружки по математике для 7 и 8 классов 29/30 октября 2013 г. Логические головоломки Введение Математика — это вовсе не запоминание формул или выполнение процедур снова и снова. Шкафчики и кошельки 4 11. Есть 9 головоломок от простых до средних и сложных. ISBN-10: 0-9819072-0-2 ISBN-13: 978-0-9819072-0-8 В этой книге рассматриваются вопросы организации, планирования и обучения класса посредством ответов на часто задаваемые вопросы. Возможно, вам не потребуется больше времени, чтобы перейти к книжному фонду с такой легкостью, как их поиск. Также разгадывай веселые загадки для старшеклассников! Дразните свой мозг этими потрясающими головоломками и шутками, которые поразят вас. Эти головоломки также можно найти в MathSphere: все решено! Загадки, головоломки и логические головоломки с ответами. Получите очевидные ответы на математические загадки. Рабочий лист. Это мероприятия в честь Дня Благодарения для старшеклассников. 17 (-20) ÷ (-4) — 7 Раскрасьте этот ответ синим цветом.Математические головоломки — забавные головоломки, охватывающие как логику, так и базовые навыки! Измерение — отличные листы для изучения базовых 10 измерений. Как отвечать на задачи по алгебре для 9 класса, включая возраст, расстояние, скорость, время и монеты, с примерами и пошаговыми решениями, Задачи по алгебре для 9 класса. а. У всех также есть ключ для ответа — в детстве я никогда не умел решать логические головоломки. Совершенствуйте математические навыки с помощью увлекательных математических головоломок. Классы K-12 соответствуют национальным и государственным стандартам.Все приведенные ниже головоломки на математическую логику являются образцами из популярного учебного пособия «101 ежедневное задание по математике для привлечения учащихся 3–8 классов» в формате PDF, которое теперь доступно! Сделайте изучение математики увлекательным и эффективным с помощью Prodigy Math Game. Решения Hotmath для учебников бесплатны и не требуют регистрационной информации. В конце подсказки иногда указывается общее количество букв, в зависимости от стиля головоломки и страны публикации. Чтобы решить эту загадку, вам не нужно вручную выполнять всю математику, а лучше попытаться выяснить закономерность.Решать проблему. У каждого кроссворда есть распечатываемый PDF-файл и печатный ключ ответа. XEARL. Этот сборник материалов для учителей для детей веселый, бесплатный и учит детей основам обучения для учащихся начальной и средней школы. Common Core Веселые шутки, загадки и головоломки FUNsheets помогают учителям, родителям, репетиторам и целеустремленным ученикам сделать изучение математики проще, веселее и сэкономить ваше время на подготовку. Math Goodies помогает детям на всех уровнях с помощью интерактивных инструкций и бесплатных ресурсов. 13 янв.2020 г. · 13 янв.2020 г. — Математические загадки с ответами pdf — Поиск в Google.(Что ж, это так, для достаточно больших значений 2. Разрежьте изображение на 2 части, из которых вы можете составить прямоугольник 6×4 квадратов. Комплексы слов — это словесные головоломки, которые описывают слово или фразу, не произнося их по буквам. LE LSWO СЕЙЧАС. И мои онлайн-пазлы, которые закончились на нашем сайте Cool Math Games, Coolmath-games. Поскольку мы предоставляем ключи для ответов, учащиеся могут самостоятельно оценивать и использовать немедленную обратную связь, полученную с помощью ключа ответа, для анализа и исправления ошибки в их работе.Дань Льюису Кэрроллу, а. Стримко, складывающийся в десять головоломок. Если вы посетите музей, то сможете увидеть этот невероятный артефакт на выставке. цукаеру. Добавляйте в свой мозг клетки и умножайте веселье с помощью забавных, простых и сложных математических загадок и ответов. В этом блоге у нас есть математические головоломки — числа, уровень 2. Ответ от 4 декабря 2019 года. 15 апреля 2020 г. · В конце этого поста также есть ссылка для загрузки PDF-файла для бесплатной печати математических логических головоломок и ключа ответов, в котором представлены все эти логические головоломки !.Имя: Добро пожаловать на сайт МАТЕМАТИКИ ДЛЯ 8-ГО КЛАССА. Для удобства читателя в разделе «Решения» мы приводим некоторых основных студентов, которые любят решать наши головоломки и головоломки по математике! У нас есть множество головоломок разного уровня сложности. У нас есть головоломки на 4 разных уровнях: уровень 1, уровень 2, уровень 3 и уровень 4. Потратив время на поиск таких терминов, как конгруэнтные, дополнительные углы, четырехугольник и радианы, ученики будут чувствовать себя менее запуганными этими терминами и более уверенно их использовать. в классе.Кроме того, в онлайн-кроссворды можно играть. Модуль 1: Разрядные и десятичные дроби: 5-й класс (Eureka Math / EngageNY) Вопросы и ответы на собеседование по логическим головоломкам для компаний-разработчиков программного обеспечения, экзамены MBA и все типы экзаменов. Геометрия — формы: создайте шестиугольник из пазла — для подготовительного и детского сада. Каков был общий доход Шармейн? _____ 2. Как только вы угадаете ответы, расшифруйте выделенные буквы, чтобы ответить на загадку (за вас было заполнено восемь букв, и буквы можно использовать несколько раз).Этот набор содержит общие математические, логические и аналитические головоломки с ответами для собеседований и экзаменов. Это логическая головоломка — это задача, задача или игра, требующая от игрока использования форм Наивысшего и наименьшего номера ISBN Digital (PDF): 9781783741441 суть вопроса или получение глупого ответа. Его следует регулярно вводить, чтобы заинтересовать молодых людей чудесами математики. Они касаются выявления закономерностей в системе уравнений и применения их к неизвестному.Они любят часами разгадывать математические загадки и головоломки. Рабочие листы Free Division без остатка и с остатком для 2-го, 3-го, 4-го и 5-го классов. Имеются рабочие листы в формате PDF для печати, головоломки для поиска слов, проверка орфографии, зашифрованные предложения, перекрестные слова и многое другое. Теперь добавьте 30. Джим (@teachmaths) бросает вызов своим ученикам старшей школы, чтобы они использовали свои математические навыки (и немного удачи) в игре «Великие эльфы». Когда они встречаются, кто был ближе к их дому? Ответ на головоломку Посетите раздел «Для учителей», где вы найдете ресурсы для учителей математики, нашу бесплатную программу KenKen Classroom и сообщение от Тэцуи Миямото, японского преподавателя, стоящего за KenKen.Числовые головоломки. Вопросы и ответы. Ответ B или ответ C Мы можем определить правильный ответ, используя дискретную математику • R (x): Ответ x правильный • Правильный ответ должен быть единственным. Ответы на эти сложные математические головоломки даны в этом видео-головоломке. Фрэнк и его брат Джек собираются встретиться друг с другом. Что живет зимой, умирает летом и растет от корней? Сосулька. Математические головоломки для старших классов. Математические головоломки — числа Уровень 2. В вашем классе ответ на простейшую головоломку работает, хотя и не обязательно сознательно, на математических принципах. 21 июля 2020 г. · Математические объяснения простым языком, а также головоломки, игры, рабочие листы и иллюстрированный словарь. Загадки и задачи из второй части этой книги можно скопировать для использования в школах Англии, участвующих в Национальной стратегии счисления. 18 (-3) (- 4) (- 4) Выделите этот ответ светло-зеленым цветом. Ясно, что это можно распространить на другие модули. Они также очень настраиваемы: вы можете контролировать количество проблем, размер шрифта, интервал, диапазон чисел и так далее. → угол площади сферы отменить центр хорды куб градус цифра эллипс увеличить равный коэффициент форма галлон дюйм индекс обратный литр мантисса мера метр минус ноль мощность секанс синус пространство сфера ноль 1.Будьте готовы уделить все свое внимание, потому что оно понадобится вам для решения этих забавных математических загадок. Эти загадки и головоломки проверят, можно ли использовать поисковые запросы Geometry, чтобы научить учащихся понимать новые и незнакомые термины. Однако сохранить это интересным иногда бывает непросто. Наклон (или градиент) линии — это число, которое обозначает «крутизну» линии, также обычно называемую «подъем за пробегом». Этот рабочий лист по математике был создан 12 декабря 2010 г. и просматривался 49 раз на этой неделе и 41 раз в этом месяце.Миямото, шахматный мастер Дэвид Леви и остальные участники команды KenKen превратили эти математические головоломки в отличные игры для обучения и тренировки мозга. Немногочисленные задачи, требующие более высокого уровня математики, отмечены (o). Как вы думаете, знаете ли вы ответ на этот вопрос. Сможете ли вы пройти этот тест по математике в начальной школе? Эти высококачественные математические рабочие листы поставляются в формате PDF и содержат ключ ответа, который прилагается к математическим рабочим листам по мере его создания. Кроме того, они призывают детей снова проверить свои логические рассуждения.Пример головоломки судоку экспертного уровня. Использование головоломок в обучении математике Жан Паркер, Центральная средняя школа, Флоренция, Алабама. 21 мая 2019 г. · 10 супер веселых математических загадок и головоломок для детей от 10 лет (включая ответы!) Сообщение Автор: Энтони Персико Некоторые из них существуют уже много веков, а другие совершенно новые, но все они были выбраны в качестве самые веселые и увлекательные математические загадки для тех, кто любит проверять свои навыки решения проблем и получать удовольствие от решения математических головоломок.кроссворды в этом разделе делятся на категории легких и сложных в зависимости от сложности слов, необходимых для выполнения. Math planet — это онлайн-ресурс, где можно бесплатно изучать математику. теперь доступен в формате PDF. Книга «Детские математические головоломки» предлагает часы развлечения. Вовлекайте детей в нашу сложную, но интересную математику Ученики, способные к математике, есть в каждой школе и среди всех этнических групп. Головоломки и задачи из второй части этой книги можно скопировать для использования учителями, чтобы убедиться, что ожидания всех учеников соответствуют Бесплатные школьные (10, 11 и 12 классы) математические вопросы и задачи с ответами. Студенты, безусловно, любят наши головоломки и головоломки по математике! У нас есть множество головоломок разного уровня сложности. С ответами Числовые головоломки Математические головоломки Математические загадки Математические игры для детей Пазлы для детей Веселая математика. Я Сара Картер, учительница математики в старшей школе, которая искренне считает, что математика — это любовь. Бросьте вызов своим детям решить эти математические головоломки на тему смайликов! Доступны различные уровни сложности с ответами для учителей! Эти математические загадки с ответами — интересный способ проверить свои способности в разгадывании математических загадок для детей; Для младших школьников; Легко; Прикольные загадки на математику Страница 1.бесплатные математические головоломки, головоломки и загадки для детей и учащихся начальной математической школы. Тем более что головоломки Бенджамина Баннекера могут быть решены учениками средних и старших классов. Предположим, что вы — заключенный, и перед вами две двери: один ведущий к свободе и один ведущий к камере палача, но рабочие листы математических головоломок pdf Коллекция карточек с заданиями по математике: задачи по математике и набор карточек для головоломок по математике A # 205985 Рабочий лист № 6 — Вот забавный раздаточный материал, полный бесплатных рабочих листов математических головоломок в формате pdf для печати, рабочих листов математических головоломок для практики и улучшения различных математических навыков, сложения, вычитания, соотношений, дробей, деления. Готовый ответ должен выглядеть так: Оса, которую он выпустил, заставила нас бежать.Наши математические головоломки и головоломки отлично подходят для использования во время уроков или в качестве математических заданий. Рабочие листы Free Division без остатка и с остатком для 2-го, 3-го, 4-го и 5-го классов Учащиеся, которые отрабатывают свои математические навыки с нашими заданиями по математике во время школьных перерывов, сохраняют свои математические навыки на высоком уровне к предстоящим учебным семестрам. pdf из MATH 101 в Staley High. Ресурсы по математике для детей, родителей и учителей для обогащения учебы. 3 класс. Удачи! Математический факультет Университета штата Мичиган 619 Red Cedar Road East Lansing, MI 48824 Телефон: (517) 353-0844 Факс: (517) 432-1562 Колледж естественных наук. Рабочие листы с логическими головоломками и рабочие листы с загадками обязательно улучшат вашего ребенка & # 039; навыки критического мышления.Эти математические загадки непросто решить. Написание чисел от 1 до 6 Игра на запоминание с картинками 6×4 Rectangle. Зарегистрироваться Сегодня! 15 апреля 2018 г. · Вы можете весело провести время, изучая математику в старшей школе. Возраст Д’Моргана 30 1 Рождественские математические головоломки 18 Найди четыре набора A — Дополнение к 20 19 Найди четыре набора B — Дополнение к 45 20 Итоговое дерево треугольников — Дополнительная головоломка 21 Три подарка — Стратегическая головоломка 23 12 дней Рождества Добавить к 12 — 24 Головоломки Лист и игра С Рождеством — 25Code Puzzle Четыре подарка — Стратегическая головоломка 27 Десять украшений — Стратегическая головоломка 28 Домашняя страница Хорхе Нуно Сильвы 29 октября 2017 г. — В каждом из этих простых вопросов математической головоломки четыре математических уравнения представлены в виде головоломки с картинками.ВЫ — главный герой своей жизни. Адаптируйте кроссворд по Алгебре 2, чтобы научить своих учеников таким понятиям, как параболы, ось симметрии, коэффициент корреляции, регрессия и мнимые числа. Формат: PDF FilesPle Класс 6 Математические кружки 10/11 октября 2017 г. Логические головоломки, головоломки и математические игры Введение Логические головоломки, головоломки и математические игры — все это может быть забавным и интересным способом бросить вызов самому себе. Итак, добавьте первое число к произведению двух чисел, чтобы получить ответ.Разрабатывайте по одной головоломке в день в качестве разминки по математике или даже для разговора с числами. 18 августа 2019 г. · Эта сложная математическая задача стала вирусной несколько лет назад после того, как она появилась на вступительном экзамене в Гонконге … для шестилетних детей. 17 августа 2016 г. · Сложные и интересные математические головоломки — только для гениальных математических головоломок с ответом! Добро пожаловать! Вот еще одна интересная математическая головоломка, которую я получил! Решите эти сложные математические загадки и прокомментируйте свой ответ. Детям необходимо знать базовые математические концепции, чтобы выполнить их, поэтому они лучше всего подходят для детей, идущих в первый класс или старше. Давайте решим несколько простых математических загадок и головоломок. . 21 октября 2014 г. · A + B = см. Вот подборка из 10 математических и логических головоломок, призванных отточить ваши умственные способности. Стратегия кроссворда: чтобы разгадать кроссворд, введите слова «Снова в школу», которые соответствуют подсказкам. Его можно распечатать, загрузить или сохранить и использовать в классе, домашней школе или другой образовательной среде, чтобы помочь кому-то Word Plexer Puzzle. Обзор математики в средней школе. В школьном отделении Форт-Ла-Боссе в Манитобе можно решать задачи по-разному, чтобы удовлетворить потребности учащихся, а также в формате Adobe PDF 24 марта 2020 г. 30 математических загадок для детей.Чтобы разобраться в схеме головоломки, потребуется некоторое время. 6 ноября 2017 г. · Математические головоломки на День Благодарения: эта бесплатная загрузка (прокрутите до конца сообщения) включает в себя 3 различных математических головоломки на тему Благодарения. и ответы Pdf 2013 | Пазлы с числами Вопросы ответы | Числовые пазлы Вопросы Ответы [Онлайн-справка по математике для старших классов и колледжей Ищете онлайн-справку по математике? У нас есть более 20 видеокурсов, которые преподают лучшие учителя, которые помогут вам быстро учиться. Результаты 1–24 из 1915 Просмотрите ресурсы по математическим головоломкам для старших классов на сайте Teachers Pay Teachers, включая лист записи ученика и ключ для ответа! Математических головоломок вообще есть, ответ на простейшую головоломку рабочий, правда не Томми Смарта недавно отправили в новую школу.Затем подсказки обозначаются этими числами и указанием направления, например, «4-поперек» или «20-вниз». Математика и практические занятия Дроби и десятичные дроби Боб Оленич Нью-Йорк • Торонто • Лондон • Окленд • Сидней Мексика Геометрические головоломки Задача 1 (но для начала, последняя логическая головоломка). 17 января 2008 г. · R Математика и не только: Книга задач Стандартная рабочая тетрадь для старших классов Первое издание Цишен Хуанг, доктор философии. Следующие темы по алгебре Электронная книга по математике: лучшие практики для начальной школы. Предложите своим ученикам творческие уроки математики, распечатанные рабочие листы, задания , викторины и многое другое в течение Месяца математического образования (апрель) — или в любое время года! Инструкции: Выберите головоломки, которые хотите добавить на рабочий лист (верхняя область). Вы можете составить числа 5034927618 и 5038167294. Сто шляп 2 2. Головоломки Стримко похожи на головоломки Судоку, но 13 апреля 2020 г. · Объединенный школьный округ долины Чино стремится предоставить всем людям равные возможности в сфере образования и трудоустройства. Математический кросс № 28 Уравнения, измерение и деньги. Что это за номер? Ответ: Семь Научитесь решать сложные математические головоломки. Было несколько разных типов иероглифов. Из второго разреза, пересекающего первый, получится четыре части, а из третьего разреза можно получить доступ к бесплатным математическим головоломкам с ответами Математические головоломки с ответами. Это также один из факторов, позволяющих получить мягкие документы этих математических головоломок с ответами в Интернете.коэффициент связи 0. Джеффри Дж. Нажмите, чтобы получить ответ на эту головоломку. Бесплатные задания по алгебре в формате pdf для скачивания, калькулятор алгебры с шагами по алгебре 6 класс, задания по алгебре 9 класс, листы по алгебре 8 класс, листы по алгебре 4-го класса, 3-й, 4-й, 5-й, 6-й, 7-й классы, учебник по алгебре pdf K-12 Math Problems , Головоломки, Советы и уловки — Математический форум Ссылки на советы по расчетам (Превзойти калькулятор, Правила делимости, Советы по умножению), наборы математических задач, а также математические числа и линейные головоломки. математика: головоломки для начинающих Ключевые слова: головоломки для начинающих Дата создания: 31.03.2004 9:49:02 AM Просмотр головоломок с дробями и десятичными числами. Ученикам может быть сложно выучить и запомнить математический словарный запас. Такие занятия, как приготовление пищи, игра в мяч и постройка чего-либо (даже сборка стула), могут быть связаны с Math Cross, Puzzle # 11. Поощряйте понимание учащимися математических терминов и повышайте их способности к математическим фактам с помощью этой забавной головоломки с крестиком. Учебник по предметам помощи в домашних заданиях. Математические загадки — это творческий способ повеселиться, а также обострить вашу проницательность с помощью чисел и деталей.Делитесь рабочими листами, учебными пособиями и словарными наборами в Google Classroom! Розеттскому камню 114. Математический крест № 27 Как узнать время. 11 + 11 = 10. Более сложные головоломки будут добавляться на регулярной основе. Давайте начнем с забавных математических головоломок! Головоломка №1: используйте четыре пятерки и некоторые символы +, ×, — и ÷, чтобы получить выражения для 0, 1, 2 и 5. Головоломка №2: в Cool Math есть бесплатные классные уроки математики онлайн, классные математические игры и развлечения математическая деятельность. Шахматная доска дьявола 4 13. Рождественские головоломки судоку. В загрузку также включены 2 головоломки судоку на рождественскую тематику.Направления: Решите каждую проблему, показывая всю работу. Некоторым из наиболее важных тем, таким как «Здоровье и болезнь» и «Неправильные глаголы», посвящено несколько головоломок, чтобы отразить их важность в лексиконе. Агент по недвижимости получил комиссию в размере 6% от продажной цены дома. Первый — это головоломка с сеткой. Они могут быть такими же простыми, как определение времени, и такими же сложными, как навыки алгебры среднего уровня. Облако тегов. Вот несколько рабочих листов по геометрии на День Благодарения. mashupmath. Этот пакет содержит задачи на повторение из предыдущих классов математики и представляет Центральную среднюю школу Вифлеема.Изучите математику в пятом классе в соответствии с учебной программой Eureka Math / EngageNY — арифметику с дробями и десятичными знаками, задачи с объемом, преобразование единиц измерения, точки на графике и многое другое. 30+ загадок для старшеклассников и ответы на их решение в 2021 году — головоломки и головоломки www. Модуль 1: Разрядные числа и десятичные дроби: 5-й класс (Eureka Math / EngageNY) Пазлы с поздравительными открытками на умножение: составьте забавную головоломку с листом приветствия для умножения, которую нужно решить. Полезно для запоминания фактов умножения. На этой странице ваши приветствия / сообщения преобразуются в арифметические задачи.Говорят, что рекреационная математика должна быть включена в стандартную учебную программу. Эта книга поможет вам √ получить высокие баллы по разделу SAT / ACT по математике, √ подготовиться к курсу по математике и √ выиграть в олимпиадах по математике в старших классах. Тысячи соискателей, готовящихся к экзаменам IBPS, SBI, All Other Bank, SSC, Railways, Insurance и другим конкурсным экзаменам, отправили запрос на предоставление головоломок высокого уровня для банковского служащего в формате PDF для подготовки к рассуждению. Только для знатоков. Даже те головоломки, которые у нас нет возможности решить, кроме заданий математических головоломок. Wanko Background NCTM повысил осведомленность о доказательствах и доказательствах, расширив стандарт процесса рассуждений, который был найден в Стандартах учебной программы и оценки (1989), до аргументов и доказательств, изложенных в Принципах и стандартах (2000). математические пазлы для старшей школы с ответами pdf

Пазлы с числами Вопросы и ответы

Домой> Рассуждение и головоломки> Числовые головоломки> Числовые головоломки Вопросы

Используя свои навыки числового и логического мышления, попробуйте выяснить, какое число отсутствует в приведенных ниже вопросах.Цифры вокруг дадут вам подсказки, необходимые для решения головоломки.



Ответ и объяснение:

Ответ : 6

Пояснение :

Если посмотреть на диаграмму в рядах, центральный круг равен половине суммы чисел в других кругах слева и справа от центра.



Ответ и объяснение:

Ответ : 9

Пояснение :

Число в центре каждого треугольника равно сумме двух нижних чисел за вычетом верхнего числа.




Ответ и объяснение:

Ответ : 19

Пояснение :

При движении по диагонали вниз числа следуют за последовательностью простых чисел.



Ответ и объяснение:

Ответ : 16

Пояснение :

Начиная с нижнего левого угла и двигаясь по часовой стрелке вокруг треугольника, числа следуют за последовательностью квадратных чисел.



Ответ и объяснение:

Ответ : 39

Пояснение :

Работая сверху вниз, удвойте каждое число и вычтите 1, затем 2, затем 3 и т. Д.

Страница 1 | Страница 2 | Стр. 3 | Стр. 4 | Стр. 5 | Стр. 6 | Стр. 7 | Стр. 8 | Стр. 9 | Стр. 10 | Стр. 11 | Стр. 12 | Стр. 13 | Стр. 14 | Стр. 15 | Стр. 16 | Стр. 17 | Стр. 18 | Стр. 19 | Page 20

Дополнительные обучающие и развлекательные тесты ниже.

Вопросы или комментарии? Пожалуйста, обсудите ниже.


Логические вопросы-головоломки с ответами

Логическое рассуждение головоломок часто используется в банковских PO, SSC, государственных экзаменах и многих вступительных тестах. В основном это головоломки на основе аранжировки, буквенно-цифровых или аналитических рассуждений. Кроме того, логические вопросы улучшают ваши дедуктивные способности и способности решать проблемы. Здесь мы собрали серию различных логических головоломок с ответами и объяснениями.Этот набор содержит 10 простых логических головоломок.

Решите заданные вопросы и проверьте свои умственные способности:

Q.1. У матери Мины пятеро дочерей: Рина, Тина, Шина и Шила. Кто пятая дочь? Решение и объяснение

Meena.

Q.2. Сколько раз вы можете вычесть число 5 из 35?

Решение и объяснение

Однажды. После первого расчета вы вычтете 5 из 30, затем 5 из 25 и так далее.26. Найдите точное числовое значение следующего уравнения: (Y — A) * (Y — B) * (Y — C) * … * (Y — Z).

Решение и объяснение

(Y — A) * (Y — B) * (Y — C) * … * (Y — Y) * (Y — Z) равно 0, поскольку (Y — Y) равно нулю.

Q. 5. У женщины было две девочки, родившиеся в одно и то же время одного дня одного года. Но они не были близнецами. Как такое могло быть?

Решение и объяснение

Это были двое из тройки.

В.6. Девушка обнаружила, что у нее есть полоска ленты шириной 56 см. Она могла отрезать сантиметр каждую секунду. Сколько времени ей потребуется, чтобы разрезать 56 частей? Она не может сложить полосу, а также не может сложить две или более полос и разрезать их вместе.

Решение и объяснение

Чтобы получить 56 штук, девушке нужно будет выложить всего 55 разрезов. то есть она может разрезать 56 штук за 55 секунд. После 54 штук у нее будет кусок длиной 2 см. Она может разрезать его на две части, сделав всего один разрез за 1 секунду. Следовательно, всего 55 секунд.

Q.7. Мужчина проехал на своей машине Innova весь путь от Шимлы до Дели и в конце поездки обнаружил, что у него с самого начала была проколота шина. Но это никак не повлияло на его машину Innova? Как это возможно?

Решение и объяснение

Проколотая шина должна быть запасной.

Q.8. У Винода была двадцать одна курица, все, кроме одиннадцати, погибли. Сколько куриц осталось у Винода?

Решение и объяснение

Одиннадцать

В.9. Мужчина строит дом со всеми четырьмя сторонами на юг. Мимо дома проходит медведь. Какого цвета медведь?

Решение и объяснение

Белый: дом построен прямо на Северном полюсе.

Q.10. Тайное агентство Tycoon отправило по электронной почте некоторый код своему агенту 006. Это «RACECAR, MURDRUM, SAGAS, ATTACK и REPAPER». Оставляя одно, которое является фальшивым, все остальные слова имеют что-то общее. Вы можете помочь найти фальшивое слово?

Решение и объяснение

Поддельное кодовое слово — АТАКА.

Все кодовые слова, кроме АТАКИ, являются палиндромами.

Логические головоломки: практические упражнения

.

Добавить комментарий

Ваш адрес email не будет опубликован. Обязательные поля помечены *